Sunteți pe pagina 1din 104

For MCQS on All subjects & Licencing examinations.

ww

w.

me

dic

alm cq s

do

wn

loa

d.b log sp

ot. co

Visit : www.medicalmcqsdownload.blogspot.com

dic

ww w. me

This Book is not published by eMedicine .It is just a selection of cases from eMedicine website by Ali Faris Haider.

alm cq sd ow

By: Dr. Ali Faris Haider

23/11/2008

nlo

From

ad .bl og

30 Clinical cases as

sp ot. c

om

3o clinical cases from eMedicine

Preface
Since the best way of learning medicine is to study medical problems

I collected some of the common cases from eMedicine website and put them in this book without editing the contents so that everyone can enjoy solving these Medical problems.

eMedicine is read by doctors and medical students from approximately 120 countries. eMedicine(www.emedicine.com) sends a weekly case via email to its subscribers.

alm cq sd ow

This book is dedicated to Junior and senior doctors. If you find this book useful; please remember me in your prayer. Dr. Ali Faris Haider 13-12-2008

ww w. me

dic

nlo

ad .bl og

What is eMedicine? eMedicine: It is web-based and consists of clinical overviews of disease entities by experts in the field. Continuously updated, mainly for professionals, over 10 000 physician authors on 7000 diseases, Articles undergoes 4 levels of physician peer review plus an addi onal review by a Pharmacy editor prior to publication, 30 000 mul media les % of radiology residents use it as ,12 the first source of information.

sp ot. c

om

3o clinical cases from eMedicine Contents


Case 1......................................................................................................................................................................... 3 Case 1 Answer............................................................................................................................................................. 5 Case 2......................................................................................................................................................................... 7 Case 2 Answer............................................................................................................................................................. 8 Case 3......................................................................................................................................................................... 9 Case 3 Answer........................................................................................................................................................... 11 Case 4....................................................................................................................................................................... 12 Case 4 Answer........................................................................................................................................................... 14 Case 5....................................................................................................................................................................... 17 Case 5 Answer........................................................................................................................................................... 18 Case 6....................................................................................................................................................................... 19 Case 6 Answer........................................................................................................................................................... 22 Case 7....................................................................................................................................................................... 24 Case 7 Answer........................................................................................................................................................... 25 Case 8....................................................................................................................................................................... 26 Case 8 Answer........................................................................................................................................................... 29 Case 9....................................................................................................................................................................... 33 Case 9 Answer........................................................................................................................................................... 35 Case 10 ..................................................................................................................................................................... 39 Case 10 Answer ......................................................................................................................................................... 40 Case 11 ..................................................................................................................................................................... 43 Case 11 Answer ......................................................................................................................................................... 44 Case 12 ..................................................................................................................................................................... 45 Case 12 Answer ......................................................................................................................................................... 47 Case 13 ..................................................................................................................................................................... 49 Case 13 Answer ......................................................................................................................................................... 50 Case 14 ..................................................................................................................................................................... 51 Case 14 Answer ......................................................................................................................................................... 52 Case 15 ..................................................................................................................................................................... 53 Case 15 Answer ......................................................................................................................................................... 54 Case 16 ..................................................................................................................................................................... 55 Case 16 Answer ......................................................................................................................................................... 56 Case 17 ..................................................................................................................................................................... 57 Case 17 Answer ......................................................................................................................................................... 58 Case 18 ..................................................................................................................................................................... 59 Case 18 Answer ......................................................................................................................................................... 60 Case 19 ..................................................................................................................................................................... 61 Case 19 Answer ......................................................................................................................................................... 62 Case 20 ..................................................................................................................................................................... 63 Case 20 Answer ......................................................................................................................................................... 64 Case 21 ..................................................................................................................................................................... 65 Case 21 Answer ......................................................................................................................................................... 66 Case 22 ..................................................................................................................................................................... 67 Case 22 Answer ......................................................................................................................................................... 68 Case 23 ..................................................................................................................................................................... 69 Case 23 Answer ......................................................................................................................................................... 72 Case 24 ..................................................................................................................................................................... 76 Case 24 Answer ......................................................................................................................................................... 78 Case 25 ..................................................................................................................................................................... 80 Case 25 Answer ......................................................................................................................................................... 82 Case 26 ..................................................................................................................................................................... 84 Case 26 Answer ......................................................................................................................................................... 86 Case 27 ..................................................................................................................................................................... 88 Case 27 Answer ......................................................................................................................................................... 90 Case 28 ..................................................................................................................................................................... 92 Case 28 Answer ......................................................................................................................................................... 94 Case 29 ..................................................................................................................................................................... 97 Case 29 Answer ......................................................................................................................................................... 99 Case 30 ....................................................................................................................................................................101 Case 30 Answer ........................................................................................................................................................102

ww w. me

dic

alm cq sd ow

nlo

ad .bl og

sp ot. c

om

3o clinical cases from eMedicine Case 1

Background

dic

On his arrival to the emergency department, the patient continued to have the sensation that his heart was racing. He denies having any chest pain, nausea, vomiting, diaphoresis, light-headedness, or recent illness. He felt well before this episode. He denies having any symptoms of infection, such as fever, cough, vomiting, diarrhea, anorexia, or dysuria. He reports increased stress at work and is drinking as many as 4 cups of coee a day. He reports no notable history of medical conditions except for a similar episode of a rapid heart rate about 4 years ago; for this, he was treated with an unknown drug for 2 years. His family history is signicant for a father who died of a myocardial infarc on at 45 years of age. The pa ent takes 1 baby aspirin daily. He denies using any over-the-counter or illicit drugs; however, he smokes 3 packs of cigare es per week. On physical examina on, the pa ent is afebrile and has a heart rate of 165 bpm and a blood pressure of 138/79 mm Hg. He appears well and is in no acute distress. Findings on head and neck examination are unremarkable. He has no jugular venous disten on. His heart rate is rapid and irregular, with an audible S1 and S2 and no gallops, rubs, or murmurs. His lungs are clear bilaterally. His abdomen is soft, nontender, and without any

ww w. me

alm cq sd ow

Figure 1

Figure 2

nlo

ad .bl og

The patient states that he was closing his shop when his heart began to beat rapidly and he had difficulty catching his breath. His symptoms started suddenly and continued when paramedics arrived minutes later. They observed a rapid heart rate on the cardiac monitor and associated rhythm strips (see Figures 1-2). He was subsequently given adenosine 6 mg en route to the hospital. The pa ent had a momentary period of asystole, but his rapid heart rate returned.

sp ot. c

A 40-year-old man presents to the emergency department (ED) with palpitations and shortness of breath that started a few minutes before his arrival.

om

RAPID HEART RATE AND SHORTNESS OF BREATH IN A 40-YEAR-OLD MAN

3o clinical cases from eMedicine


masses. He has no peripheral edema. Results of his laboratory workup, including a CBC, serum electrolyte and cardiac enzyme measurements, and a coagulation panel, are all normal. His chest radiograph is also normal.

An ECG is obtained (see Figure 3).

What is the diagnosis?

Hint: Paramedics gave the patient a single dose of adenosine, which had no lasting effect on the rapid rhythm. o Ventricular fibrillation o Atrial fibrillation o Torsade de pointes o Sinus tachycardia

ww w. me

dic

alm cq sd ow

Figure 3

nlo

ad .bl og

sp ot. c

om

3o clinical cases from eMedicine o Case 1 Answer

Discussion:

The ECG shows an irregular heart rhythm with no discernible P waves. Rapid atrial fibrillation (AF) may be hard to differentiate from a narrow supraventricular tachycardia (SVT) without close examination of an ECG. The two conditions can result in similar symptoms of heart palpitations and shortness of breath. Patients with rapid AF palpitations are not uncommonly given adenosine to treat presumed SVT, as in this case. Although this treatment is typically unsuccessful, the underlying atrial rhythm may be accurately determined when the heart rate briefly slows. The conversion from a normal sinus rhythm to AF may be due to a number of conditions, including hyperthyroidism, anemia, infection, ischemic heart disease, valvular disease, drug intoxication, or use of stimulants. Increased stress and overconsumption of coffee are likely to have been the instigating factors in this overconsumption patient. AF is a common arrhythmia characterized by chaotic atrial depolarizations without effective atrial contractions. This rhythm is often seen with increasing age, with a male predominance. This arrhythmia can result in male decreased cardiac output and the formation of atrial thrombi. Many patients with AF are asymptomatic, and most have recurrent episodes without knowledge of them. The American College of Cardiology established a classification system for AF that is based on its duration and etiology. The categories are paroxysmal AF, persistent AF, permanent AF, and lone AF. In paroxysmal AF, the episodes last less than 1 week. If they recur, the condi on is considered recurren paroxysmal AF. In persistent t AF, the episodes last longer than 1 week. In permanent AF, the episode lasts longer than 1 year without any attempts for conversion or with attempts that fail. Finally, in lone AF, no underlying structural cardiac or pulmonary disease is found. Patients with lone AF have a low risk of mortality and thromboembolism and may y have paroxysmal, persistent, or permanent AF.

The patient's history should include the time of onset, the frequency of episodes, any associated symptoms, and any history of treatment for AF. Laboratory studies may be useful in determining possible etiologies of AF. The ment WBC count may help in finding an underlying infection, and the hemoglobin concentration may demonstrate anemia. Electrolyte levels, such as magnesium and potassium levels, may be abnormal, and an elevated creatinine value may indicate a renal insufficiency. Certain illicit drugs can cause a rapid heart rate; therefore, a toxicology screening may be useful when indicated. Hyperthyroidism can predispose patients to AF. For this reason, an evaluation of thyroid function with measurement of the patient's thyroid thyroid-stimulating hormone (TSH) level is warranted. AF can be diagnosed when the ECG shows an irregular rhythm with the absence of P waves. In addition, examine the patient for any signs of left ventricular hypertrophy, bundle branch blocks, and atrioventricular ient

ww w. me

The workup for AF involves careful history taking and physical examination, laboratory studies (including a CBC CBC, serum electrolyte tests, toxicology screening, and thyroid function tests), ECG, chest radiography, and echocardiography.

dic

alm cq sd ow

nlo

ad .bl og

Diagnosis: Atrial fibrillation (AF) with a rapid ventricular response. :

sp ot. c

om

RAPID HEART R ATE AND SHORTNESS OF BREATH IN A 40-YEAR -OLD MAN

3o clinical cases from eMedicine


(AV) nodal blocks, as well as for evidence of cardiac ischemia or previous myocardial infarction. Chest radiographs may be useful in evaluating the cardiac silhouette for cardiomegaly and the lung fields and vasculature for evidence of airspace disease or pulmonary edema. A transthoracic echocardiogram can be obtained to identify the size and motion of the atria, ventricles, and cardiac valves, and it can reveal pericardial disease. Transesophageal echocardiography is more sensitive than transthoracic echocardiography for diagnosing left atrial thrombus or left atrial appendage thrombus. Rate control is important in pa ents who present with rapid AF of more than 72 hours' dura on, and beta blockers (metoprolol or atenolol) or calcium channel blockers (verapamil or diltiazem) are recommended in patients who do not have an accessory pathway. Digoxin and amiodarone are the drugs of choice for controlling rapid AF in patients with left ventricular failure and no accessory pathway; however, digoxin should be loaded over 24 hours. Therefore, it is unlikely to have a notable eect in the acute se ng. If unable to achieve rate control with pharmacologic therapy, catheter-directed AV nodal ablation by a cardiologist may be considered. Anticoagulation treatment is also recommended for most patients with persistent AF, and it is typically achieved with warfarin (dosed to maintain an interna onal normalized ra o [INR] of 2.0 In pa ents considered to be -3.0). at low risk for thromboembolism or in patients who have a contraindication to the use of warfarin, aspirin can be given instead. Conversion to a sinus rhythm may be achieved with pharmacologic agents or with synchronized external electrical cardioversion. Conversion should be done only when the risk of thromboembolism is limited, as in pa ents with an onset of symptoms less than 72 hours before presenta on, in those who received an coagula on for 3 weeks, or in thos whom transesophageal echocardiographic results rule out a left atrial e in thrombus. A er successful cardioversion, an coagula on therapy should con nue for at least 1 month to decrease the risk of thromboembolism, which may occur from the formation of a mural thrombus. After cardioversion is done and the patient's AF reverts to a sinus rhythm, use of daily outpatient antiarrhythmic drugs is not typically recommended, as these drugs have associated risks; therefore, they should be taken only when patients have persistent or frequently recurring symptoms. Antiarrhythmic drugs that can be used to convert AF to a normal sinus rhythm include ibutilide, flecainide, procainamide, and amiodarone. Each has different risks, success rates, and indications based on the duration of AF. As a group, an arrhythmic drugs can convert 3060% of cases of AF to a normal sinus rhythm. Electrical cardioversion has a higher success rate, conver ng 7595% of AFs to normal sinus rhythms. Electrical cardioversion may be done in a nonemergency se ng a er 3 weeks of an coagula on treatment to decrease the risk of thromboembolism, or it may be required on an emergency basis in a hemodynamically unstable patient. In this situation, AF often has an acute onset, and the benefits of cardioversion outweigh the risks of thromboembolism. The role of cardioversion to manage AF in the emergency department is an emerging one. Patients who are at low risk, who are clinically stable, and who present to the emergency department with new-onset AF can be treated with chemical or electrical cardioversion and safely discharged, home with close follow-up, by a primary physician or cardiologist.

ww w. me

dic

alm cq sd ow

nlo

ad .bl og

sp ot. c

om

3o clinical cases from eMedicine Case 2

Background

Hint The pa

ent looks comfortable despite changes on his ECG. He had symptoms of a common cold 3 weeks ago.

ww w. me

dic

alm cq sd ow

Figure 1

nlo

ad .bl og

sp ot. c

A 38-year-old man without cardiac risk factors presents with chest pain that has been con nuous for 2 days. His vital signs are as follows: temperature, 99F; heart rate, 86 beats per minute; blood pressure, 140/78 mm Hg; and respiratory rate, 21 per minute .

om

CHEST PAIN FOR 2 DAYS

3o clinical cases from eMedicine Case 2 Answer

Diagnosis: Acute pericarditis Discussion:

The ECG shows a normal sinus rhythm (NSR) that is regular, with normal intervals. ST elevation is notable in leads II, III, aVF, and V3 through V6. Note that the PR depression is most visible in leads II and V2 through V6. Corresponding PR elevation is noted in the aVR lead .

ww w. me

dic

alm cq sd ow

nlo

ad .bl og

The ECG findings are consistent with acute pericarditis, which can cause friction rub to be heard on examina on. In stage I (as shown here), diuse ST eleva on is present in leads I, II, aVL, aVF, and V3 through V6, and reciprocal ST depressions are seen in leads aVR and V1. Inamma on cannot occur supercial to the thin walled atria; therefore, some myocarditis occurs. Exaggerated atrial T waves cause PR-segment depressions with a polarity opposite to that of the P wave. PR elevation, as shown in the aVR lead, is a reliable indicator of pericarditis. In stage II, the ST elevations and PR depressions resolve, and the ECG normalizes. In stage III, generalized T-wave inversions are present in most or all leads. In stage IV, the ECG returns to its state before the pericarditis, or the T-wave inversions become permanent.

sp ot. c

om

CHEST PAIN FOR 2 DAYS

3o clinical cases from eMedicine Case 3

Background

ww w. me

dic

For the rst 2 days of this illness, the pa ent had a mild runny nose, a sore throat, and some diarrhea, all of which were self-limited. Since his illness began, he has had an on-and-off headache, in addition to increasing weakness and anorexia. On systemic review, the findings are otherwise essentially negative, including the

alm cq sd ow

nlo

Figure 1

ad .bl og

sp ot. c

A 25-year-old fourth-year Filipino medical student presents to a California walk-in clinic with an intermittent fever, chills, and cough las ng almost 5 days. He has had progressively worsening muscle aches. Over the last 48 hours, he has noticed increasing difficulty in taking a deep breath. Because the patient has had no prior medical illnesses and only occasional colds, he did not seek medical care earlier .

om

FEBRILE PNEUMONIA IN A MEDICAL STUDENT RETURNING FROM SOUTHEAST ASIA

3o clinical cases from eMedicine


absence of a rash, headache, abdominal pain, vomiting, and back pain. The patient smokes less than a pack of cigarettes per day but does not have chronic bronchitis. He denies using alcohol, illicit drugs, and prescription medications . The pa ent is nishing an 8week hospital-based rotation in Southeast Asia in which he was an acting intern. He wanted to study internal medicine and critical care medicine in the region because he aspires to work in interna onal health. During his rota on, he was in Manila for 4 weeks and then Singapore for 2 weeks. A week and a half ago, he returned to the United States to nish his nal 2 weeks at the hospital associated with his California medical school . The patient reports that, during his elective, several nursing staff had been sick with respiratory symptoms "at the same time" and that the morale was poor among those who were relatively well. On further questioning, the patient states that he had direct contact with several patients in the ICU and emergency department. Additionally, he recalled escorting several septic patients to the radiology department for their imaging studies . The pa ent is 5 9 in tall and weighs 155 lb (70.3 kg) appears apprehensive and acutely ill, with a cough but . He no sputum. His vital signs are as follows: temperature, 101.3F (38.5C) on admission to the emergency department; blood pressure, 110 mm Hg systolic, 65 mm Hg diastolic; heart rate, 108 beats per minute; and respiratory rate, 18 breaths per minute with no retrac ons. The pa ent's mental status is normal with a nonfocal neurologic examination; he has no meningismus. The patient's mucous membranes are dry, and he has a few petechiae on his upper palate and no pharyngeal exudates. The patient has diffuse bilateral rhonchi with few bibasilar rales. Slight splinting is observed on both sides when the patient is asked to take a deep breath. No murmur or gallop is noted. The patient has no edema or rash in the extremities. Examination findings of the abdominal, genitourinary, and musculoskeletal areas are negative .

Hint:

This is an acute, febrile, progressively worsening respiratory illness. Epidemiologic clues include the patients recent international travel and exposure to a hospital health care worker.

ww w. me

dic

alm cq sd ow

CBC ndings are as follows: WBC count, 2.5 X 109/L (no le shi ); lymphocyte count (LYC), 0.8 X 109/L; hemoglobin (Hgb), 11 mg/dL; hematocrit (Hct), 33%; and platelet count, 89,000 per cubic milliliter. Chemistry ndings are as follows: sodium, 131 mg/dL; BUN, 35 mg/dL; crea nine (Cr), 1.6 mg/dL; aspartate aminotransferase (AST), 73 mg/dL; alanine aminotransferase (ALT), 65 mg/dL; lactate dehydrogenase (LDH), 397 mg/dL; and O2 satura on, 89%. ABG measurements are pH, 7.32; pCO2, 31 mm Hg; and pO2, 56 mm Hg .

nlo

ad .bl og

sp ot. c

om

3o clinical cases from eMedicine Case 3 Answer

Diagnosis: Severe acute respiratory syndrome (SARS) Discussion:

Severe acute respiratory syndrome (SARS) is probable on the basis of both the clinical and the radiographic criteria. (Note that this working diagnosis is based on the features of the case definition used in the SARS outbreak, ie, syndromic definition, rather than on confirmed laboratory results. Laboratory evidence is not required to meet the criteria for defining a case of SARS). The chest radiograph depicts bibasilar interstitial infiltrates and a hazy appearance in both lobes. No pleural effusion or cavitation is seen. The chest CT scan depicts patchy ground-glass opacification of both lower lobes that is most prominent in the retrocardiac region. Although several features of this case support a range of differential diagnoses, this scenario includes some of the distinctive characteristics of the newly described disease, SARS. The most important epidemiologic feature is travel to one of the main countries known to have documented or suspected community transmission of SARS, namely, Singapore (as of February 1, 2003). Other noteworthy features include the initial atypical pneumonia with a fever, myalgias, and a dry cough, followed by increasing lower respiratory symptoms and signs within an illness period of about 1 week. This illness also seemed to occur a er a 2 to 10-day incubation period, which is considered the incubation period for SARS . As a health care worker, the patient likely had direct exposure to the respiratory secretions of infected patients in Singapore. Additionally, he had diarrhea, a less common feature. Although this finding may dissuade clinicians form diagnosing SARS, it is noted in as many as 10% of cases. The pa ent^s leukopenia, lymphopenia, and elevated LDH level, along with hyponatremia and an elevated ALT level, also support the working diagnosis . Given the patient^s stable ini al course for the rst 23 days, followed by worsening ndings in the lower respiratory tract, the course is consistent with the natural history of SARS observed thus far. The infectioncontrol practices that may have been in place during the student^s rotation are unclear, and appropriate protection protocols, including use of N-95 masks, universal precau onary measures, respiratory protec on, and negative isolation, may not have been in use. Among other staff, medical students may take greater personal risk in caring for patients because of their idealism and altruism . This patient smokes, and factors such as older age and comorbid conditions (eg, smoking or diabetes mellitus) may increase a patient^s risk of severe disease progression. The radiographic features are typical for the worsening SARS presentation. With an obvious A-a gradient, the incipient picture is that of an ominous challenge in air-exchange resulting in shunt physiology. Laboratory tests to confirm the presence of the SARScoronavirus are available. Serologic tests must be performed on an acute and convalescent basis (>21 d a er the onset of symptoms). At the me of this pa ent^s presenta on, which was only-14 days postexposure 10 and less than 1 week into the course of natural illness, the enzyme-linked immunosorbent assay (ELISA) results would likely have been falsely negative and certainly not adequate to rule out coronavirus-associated SARS.

ww w. me

dic

alm cq sd ow

nlo

ad .bl og

sp ot. c

om

FEBRILE PNEUMONIA IN A MEDICAL STUDENT RETURNING F ROM SOUTHEAST ASIA

3o clinical cases from eMedicine Case 4

dic

A panel of preoperative blood tests and an electrocardiogram (ECG) are ordered. While the patient is waiting in the preoperative holding area, he experiences an episode of lightheadedness. Upon noting a rapid pulse, a technician attaches leads to obtain a cardiac rhythm strip and an ECG. His blood pressure is recorded at 80/46 mm Hg. A 12-lead ECG is obtained (see Figure 1).

ww w. me

On physical examination, the patient is afebrile, with a blood pressure of 125/67 mm Hg, a pulse of 75 bpm, a respiratory rate of 20 breaths/min, and an oxygen satura on of 91% while breathing room air. The pa ent appears younger than his stated age. He is in no acute distress. The neck examination shows no appreciable jugular venous distension or carotid bruits. His heart sounds are remarkable for a regular heart rhythm, with frequent skipped beats and a slightly accentuated second heart sound. No murmurs, rubs, or gallops are appreciated. Auscultation of his chest reveals distant breath sounds with no wheezing, crackles, or rhonchi. There is no peripheral edema of the lower extremities. The remainder of the physical examination is unremarkable.

alm cq sd ow

Figure 1

nlo

ad .bl og

Background An 80-year-old man is referred for a preoperative cardiac risk assessment prior to elective surgery for a hernia repair. He has no personal history of coronary artery disease or myocardial infarction, and no known family history of atherosclerotic heart disease. The patient denies having any symptoms of episodic chest pain and pressure, palpitations, nausea, vomiting, diaphoresis, or syncope. He does, however, admit to some exertional shortness of breath that he attributes to a long history of habitual cigarette smoking, as well as an established diagnosis of chronic obstructive pulmonary disease (COPD). In addition to the shortness of breath, he has also experienced brief episodes of lightheadedness from time to time. The lightheadedness occurs without warning and without any identifiable precipitating factor, and it abates without intervention. Other than his COPD, the patient has been remarkably healthy his whole life. His review of systems, other than as noted above, is negative.

sp ot. c

om

LIGHTHEADEDNESS IN AN ELDERLY MAN

3o clinical cases from eMedicine


What rhythm does the first portion of the ECG demonstrate? Hint: Note the association between the P waves and each QRS complex. o o o o Nonsustained ventricular tachycardia Supraventricular tachycardia (SVT) with aberrant conduction Atrial Flutter Torsade de Pointes

ww w. me

dic

alm cq sd ow

nlo

ad .bl og

sp ot. c

om

3o clinical cases from eMedicine o Case 4 Answer

Discussion:

ww w. me

dic

Ventricular tachycardia is the most common cause of wide complex tachycardias, accounting for as many as wide-complex 80% of cases. The frequency can be even higher in pa ents with structural or ischemic heart disease. Ventricular tachycardia also occurs in patients with electrolyte abnormalities, such as hypokalemia and hypomagnesemia, as well as in hypoxemic patients, individuals with acidemia, and patients with mitral va valve prolapse. The rhythm can occasionally occur in individuals without any identifiable risk factors. Adverse drug reactions can also induce ventricular tachycardia by prolonging the QT interval. Drugs that are known to

alm cq sd ow

Figure 1

nlo

A wide-complex tachycardia is a cardiac dysrhythmia with a ventricular rate that exceeds 100 bpm in the se ng complex of a QRS dura on greater than or equal to 120 milliseconds. A widecomplex tachycardia can originate from either a ventricular focus or a supraventricular focus associated with a conduction abnor abnormality. In this case, based on the QRS morphology (the QRS width being greater than 140 milliseconds at the widest leads) and the atrioventricular dissociation (see arrows), the ECG was determined to have the characteristics of ventricular tachycardia (see subsequent discussion on how to determine the focus of a wide complex tachycardia). wide-complex

ad .bl og

Diagnosis: Nonsustained ventricular tachycardia

sp ot. c

om

LIGHTHEADEDNESS IN AN ELDERLY MAN

3o clinical cases from eMedicine


increase the risk for ventricular tachycardia include digitalis, phenothiazines, tricyclic antidepressants, some long-acting antihistamines, and, paradoxically, antiarrhythmics. Digitalis toxicity can also cause a rare bidirectional ventricular tachycardia in which the QRS complexes in any given lead alternate in polarity. Lastly, common iatrogenic causes of wide-complex tachycardia in certain settings are electronic pacemakers or implantable cardioverter-defibrillators (ICDs) with pacemaker capability.

Several studies have, by utilizing various criteria or combinations of criteria, attempted to improve the diagnostic accuracy of differentiating ventricular tachycardia from supraventricular tachycardia in the evaluation of wide-complex tachycardia. Although no single algorithm is 100% sensi ve and 100% specic, several characteristics and clues can be of use. One of the most well recognized systematic algorithms consists of 4 dieren a ng characteris cs proposed by Brugada, and it is described as follows: If an RS complex cannot be identified in any precordial lead, ventricular tachycardia can be diagnosed with 100% specicity and 21% sensi vity. If an RS complex is clearly distinguished in one or more precordial leads, the interval between the onset of the R wave and the deepest part of the S wave (RS interval) is measured (if RS complexes are present in several precordial leads, the longest RS interval is used). If the RS interval is greater than 100 milliseconds, ventricular tachycardia can be diagnosed with 98% specicity and 66% sensi vity. If the RS interval is less than 100 milliseconds, the presence or absence of atrioventricular (AV) dissociation must be determined. Evidence of AV dissocia on is 100% specic and 82% sensi ve for ventricular tachycardia; this is because atrioventricular dissociation does not occur in supraventricular tachycardia. AV dissociation is characterized by atrial activity that is completely independent of ventricular activity. Although the presence of AV dissociation establishes ventricular tachycardia as the etiology, its absence does not exclude the possibility of ventricular tachycardia with retrograde conduction of ventricular impulses through the AV node producing an atrial rhythm. This phenomenon, called retrograde ventriculoatrial conduction, is easily misinterpreted as atrioventricular conduction because of the presence of P waves.

ww w. me

dic

alm cq sd ow

Accurately diagnosing the underlying rhythm in a patient with a wide-complex tachycardia is critical for determining treatment and management, especially if the patient presents emergently and is hemodynamically unstable. This is of particular concern because medications routinely used to treat supraventricular tachycardia can cause severe hemodynamic deterioration by inducing the relatively stable rhythm of ventricular tachycardia to degenerate into ventricular fibrillation, with subsequent cardiac arrest. In fact, misdiagnosis of ventricular tachycardia as supraventricular tachycardia with abnormal conduction in patients presenting with a widecomplex tachycardia is not uncommon, especially if the abnormal rhythm is hemodynamically tolerated. In general, if the clinician is unsure, a wide-complex tachycardia should be presumed to be a ventricular tachycardia until the presence of supraventricular tachycardia can be definitively proven. A patient with a widecomplex tachycardia in an unstable condition should receive immediate electrical cardioversion. In patients with a stable ventricular tachycardia or with a wide-complex tachycardia of unclear origin, pharmacologic agents, including amiodarone, procainamide, or lidocaine, may be used in accordance with established Advance Cardiovascular Life Support (ACLS) guidelines. If a determination is made that a wide-complex tachycardia is a supraventricular tachycardia with abnormal conduction, a trial of vagal stimulation (carotid massage) or treatment with adenosine may be attempted.

nlo

ad .bl og

The symptoms associated with a wide-complex tachycardia are typically caused by decreased cardiac output, and they include orthostasis, hypotension, presyncope, syncope, dyspnea, and exercise limitation. Interestingly, monomorphic ventricular tachycardia can be asymptomatic, despite the widespread belief that sustained ventricular tachycardia always produces symptoms. Clinical symptomatology is, therefore, of limited utility in the differentiation of ventricular tachycardia from supraventricular tachycardia.

sp ot. c

As mentioned above, a tachycardic rhythm with a wide QRS complex can also occur in association with a supraventricular tachycardia with abnormal conduction, which can make differentiation of ventricular tachycardia from supraventricular tachycardia with aberrant conduction difficult in the acute setting (especially since both types of patients may present with similar symptoms).

om

3o clinical cases from eMedicine


If the RS interval is less than 100 milliseconds, and if atrioventricular dissocia on cannot clearly be demonstrated, the QRS morphology may be evaluated. Morphologic criteria suggestive of ventricular tachycardia are extensive and complex, and they should be evaluated in conjunction with a cardiologist, if necessary. This nal step in evalua on moves the algorithm to 96.5% specic and 98.7% sensi ve. Other characteristic ECG findings that can be helpful for quickly differentiating ventricular tachycardia from supraventricular tachycardia include the following: An extreme rightward axis (-90 to -180 degrees) is o en more sugges ve of ventricular tachycardia. A slight irregularity of the RR intervals, especially in the early stages before settling into a regular rhythm, can be suggestive of ventricular tachycardia. The width of the QRS complex can also be useful for distinguishing supraventricular tachycardia from ventricular tachycardia. In general, a wide QRS complex greater than 140 milliseconds suggests tachycardia; however, a QRS dura on of less than 140 milliseconds is not helpful for excluding ventricular tachycardia, because ventricular tachycardia is sometimes associated with a relatively narrow QRS complex. If the degree of voltage change in the rst 40 milliseconds of the QRS complex is less than the degree of voltage change in the last 40 milliseconds of the complex, this nding is sugges ve of ventricular tachycardia. "Fusion" occurs when a supraventricular impulse reaches the atrioventricular node simultaneously with a ventricular impulse. The resulting QRS complex has a hybrid morphology that is between a narrow atrial complex and a wide ventricular complex. Intermittent fusion beats during a wide-complex tachycardia indicate atrioventricular dissociation and, therefore, also indicate ventricular tachycardia. A "capture beat" occurs when a supraventricular rhythm briefly conducts in a normal fashion, with a resultant normal QRS complex. The term "capture beat" implies that the normal conduction system has momentarily replaced the control of a ventricular focus; hence, ventricular tachycardia is present.

As mentioned above, the patient in this case was diagnosed with ventricular tachycardia, and his elective surgery for repairing the hernia was put on hold. An electrophysiology study was arranged after consultation with a cardiologist. An arrhythmogenic focus of myocardial irritability, which was thought to be caused by scar tissue from an unrecognized previous myocardial infarction, was identified during the study. The patient had an automatic internal cardiac defibrillator placed and, subsequently, his hernia was successfully repaired.

ww w. me

dic

alm cq sd ow

nlo

ad .bl og

sp ot. c

om

3o clinical cases from eMedicine Case 5

Background

Hint

Physicians in ancient Greece called this illness phthisis.

ww w. me

dic

alm cq sd ow

Figure 1

nlo

ad .bl og

sp ot. c

A 19-year-old man presents to the hospital with complaints of fever, a productive cough, night sweats, anorexia, and weight loss. He recently immigrated to Canada from Middle east. What is the diagnosis?

om

MAN FROM MIDDLE EAST

3o clinical cases from eMedicine Case 5 Answer

Diagnosis: Postprimary cavitary tuberculosis Discussion:

Postprimary tuberculosis tends to localize in the apical and posterior segments of the upper lobes (85%) and in the superior segments of the lower lobes (10%). Chest radiographs demonstrate cavita ons in 2045% of patients in whom the infection is reactivated.

ww w. me

dic

alm cq sd ow

nlo

ad .bl og

sp ot. c

om

MAN FROM MIDDLE EAST

3o clinical cases from eMedicine Case 6

Background

ww w. me

dic

alm cq sd ow

nlo

Figure 1

ad .bl og

A 47-year-old man presents to the emergency department (ED) with a chief complaint of waking up with severe abdominal pain. He states that over the past week he has had intermittent, gaslike epigastric pain and a sensation "like I need to burp"; as of this morning, the pain has acutely worsened. It is now radiating in a bandlike pattern throughout the patient's upper abdomen and to his back. The pain is most intense when he lays flat on his back and seems to be slightly better when he is sitting upright. The patient has mild nausea but has not vomited. He has not experienced any chills or fever, and he denies having any diarrhea (although the patient did have one nonbloody bowel movement before coming to the ED). On the review of symptoms, no chest pain, shortness of breath, or palpitations are noted. Other than a history of alcohol use (the patient drinks approximately 6-8 beers daily), he has no chronic medical condi ons and does not take any medica ons. He denies using over-the-counter pain medications, including nonsteroidal anti-inflammatory drugs (NSAIDs). He does not have any medication allergies.

sp ot. c

om

A 47-YEAR-OLD MAN WITH ACUTE EPIGASTRIC PAIN

3o clinical cases from eMedicine


On physical examination, the patient is noted to be a thin, slightly emaciated man who appears uncomfortable and in obvious distress. His vital signs include a temperature of 95.7F (35.4C), a pulse of 87 bpm, a respiratory rate of 28 breaths/min, a blood pressure of 111/62 mm Hg, and an oxygen satura on of 98% while breathing room air. He is diaphoretic and is writhing around in his gurney. The patient's sclerae are anicteric. The oropharynx is clear, with slightly dry mucous membranes. The heart examination reveals a regular rhythm, with no murmurs. The lungs are clear to auscultation in all fields, and no rales or rhonchi are found. The abdominal examination is notable for exquisite tenderness in the epigastric and bilateral upper quadrant regions, with focal rebound tenderness and guarding. No tenderness or palpable masses are found in the patient's lower abdomen. The rectal examination reveals heme-negative, brown stool.

ww w. me

dic

The pa ent is urgently placed on a cardiac monitor, and an 18guage peripheral intravenous (IV) line is inserted into the antecubital fossa, through which infusion of normal saline is ini ated. The pa ent is given 2 doses of IV hydromorphone, without significant improvement in his pain or abdominal tenderness. An upright, portable anterior/posterior chest radiograph is obtained, and it is noted to appear normal, with no air visualized under the diaphragm. An abdominal ultrasonogram is taken that shows no evidence of gallstones or biliary wall

alm cq sd ow

Figure 2

nlo

ad .bl og

sp ot. c

om

3o clinical cases from eMedicine


thickening; the kidneys and liver appear normal as well. All laboratory investigations, including a complete blood count (CBC), metabolic panel, hepatic panel with lipase, and troponin, are within normal limits. An electrocardiogram (ECG) is taken that reveals a normal sinus rhythm at a ventricular rate of 88 bpm, with nonspecific ST flattening in the lateral leads without any change from his prior ECG. After the initial workup is completed, an additional dose of IV pain medication is administered to the patient, which provides some relief from his pain (although focal epigastric tenderness to palpation persists). The patient is prepped for a computed tomography (CT) scan of the abdomen and pelvis, and oral and IV contrast images are obtained (see Figures 1 and 2).

What is the cause of the patient's acute abdominal pain? Hint: The acute event is likely the result of an underlying pathology. o o o o Gastroesophageal reflux disease Acalculous cholecystitis Acute pancreatitis Perforated peptic ulcer

ww w. me

dic

alm cq sd ow

nlo

ad .bl og

sp ot. c

om

3o clinical cases from eMedicine o Case 6 Answer

Diagnosis: Perforated peptic ulcer Discussion:

Transverse cuts obtained from the CT scan of the abdomen and pelvis (see Figures 1 and 2) showed free air underneath the diaphragm consistent with a perforated viscous. The images also demonstrated fluid in the region of the distal antrum/pylorus, with a small pocket of air in this fluid, suggesting that the stomach was the site of the perforation. The patient's history of alcohol use pointed to a diagnosis of a perforated gastric ulcer. 's Regarding a potential differential diagnosis in epigastric abdominal pain, several life life-threatening etiologies that must be recognized and treated urgently are possible. Cardiovascular etiologies, including acute coronary Cardiovascular syndrome and aortic dissection, must be considered, even when frank chest pain is absent. Numerous gastrointestinal causes can present in a very similar fashion. Most commonly, a relatively benign cause, such a as mild esophagitis and gastritis, is responsible. The pain of an uncomplicated peptic ulcer is comparable to that of a perforated peptic ulcer, although it is typically chronic in nature. Gallbladder disease ranges from relatively mild biliary colic to acute cholecystitis. Liver diseases include acute hepatitis; masses, such as abscesses or te tumors; gonococcal or chlamydial perihepatitis (Fitz (Fitz-Hugh-Curtis syndrome) in women; and acute cholangitis. Curtis Acute pancreatitis may be present, with or without the presence of gallbladder disease. Acute appendicitis may presence first present with upper abdominal or midabdominal pain before localizing to the right lower quadrant. Pulmonary processes, such as pneumonia, must also be considered in patients complaining of upper abdomi abdominal pain, even in the absence of cough or shortness of breath. With such a broad differential diagnosis in the presentation of epigastric abdominal pain, the workup (including laboratory investigations and radiologic imaging) has to be individualized based on age and other risk factors for each potential disease process and on the characteristics and associated symptoms of the pain. Additionally, repeat assessment of the symptoms and physical examination should be performed during the course of the evaluation. The administration of parenteral pain medication will often enable localization of the source of pain on. and assessment of the severity of the disease; pain medication should not be withheld for fear of "masking" a potentially serious disease process. As illustrated in this case, the patient had a somewhat vague examination with an essentially normal initial workup, including no evidence of perforation on an upright radiograph at presentation; this may have been dismissed as "benign" pain if it were not for the persistence of pain and for tenderness despite the administration of pain medication. Uncomplicated peptic ulcer disease (PUD) is highly prevalent in the United States. When combined with duodenal ulcers, the incidence is 1.8%, or approximately 500,000 new cases annually. Additionally, there are new about 4 million recurrences yearly. Approximately 90% of duodenal ulcers and 75% of gastric ulcers are associated with Helicobacter pylori infection. Although it is still unclear, H. pylori appears to cause injury to the stomach and duodenum through 3 poten al mechanisms, including the produc on of toxins that cause local tissue injury, the induction of a mucosal immune response, and the increase of gastrin levels with an increase in acid secretion. After H. pylori infection, NSAIDs are the most common cause of PUD. The risk of disease and ri complications (such as hemorrhage or perforation) are proportional to the daily dose taken. Advanced age and concurrent use of anticoagulants or steroids also increase the risk for complication. Other factors that may predispose a patient to gastric ulceration include chronic alcohol intake, smoking, and infection.

ww w. me

dic

alm cq sd ow

nlo

ad .bl og

sp ot. c

om

A 47-YEAR -OLD MAN WITH ACUTE EPIGASTRIC PAIN

3o clinical cases from eMedicine


Perforation of a peptic or duodenal ulcer into the peritoneal cavity has the potential for significant morbidity and mortality. The majority of cases occur in the elderly and in persons using NSAIDs. It is not uncommon that in cases of perforated peptic ulcer, low-dose aspirin is the only NSAID taken. Remarkably, smoking appears to be a stronger risk factor in patients under the age of 75 than NSAIDs are. Although most pa ents will give a history of chronic epigastric pain prior to perforation, caused by the presence of the uncomplicated ulcer itself, 10-25% of pa ents will have no symptoms leading up to the perfora on. In approximately 10% of pa ents, the perforation is accompanied by hemorrhage. Perforated peptic ulcers are most often located at the lesser curvature of the stomach. The majority of perforated duodenal ulcers involve the anterior wall of the duodenal bulb. Three classic clinical stages typically occur in patients with perforated ulcers. The first stage, caused by the rapid release of gastric juice into the peritoneal cavity, is characterized by an abrupt onset of intense abdominal pain. The duration and intensity of this stage varies based on the size of the perforation and the extent to which gastric juice leaks into the peritoneal cavity. During the second stage, which often does not occur, there is spontaneous improvement in the symptoms. This likely happens as a result of fluid pouring out of the injured intraperitoneal tissues, which causes a buffering of the acidic gastric juice. It is important to recognize that this stage may occur; the clinician should not feel overly reassured if this occurs in a patient with an otherwise concerning presentation. Additionally, findings of peritonitis will typically still be present despite improvement in the patient's symptoms. The final stage of frank peritonitis is characterized by increased pain and signs of a systemic inflammatory response. If appropriate therapy is not initiated, death soon follows. As demonstrated in this case, the absence of free air within the abdominal cavity on a plain, upright abdominal or chest radiograph is not noted in all cases. In fact, evidence of free air with a perforated viscous is seen in only approximately 30% of cases. CT scanning has a much higher sensi vity for visualizing free air and may show evidence suggesting the location of the perforation. Endoscopy must be avoided if a perforation is suspected, since air insufflation in the stomach may open a perforation that has sealed, resulting in increased leakage of gastric juice into the peritoneum. As an alternative to endoscopy, an upper gastrointestinal series using a watersoluble contrast agent may be performed. A perforated ulcer should initially be treated with adequate resuscitation, which may include airway management (if indicated), the administration of adequate fluids (especially if signs of hypovolemia or a systemic inflammatory response, such as hypotension and/or tachycardia, are present), and pain control. Broad-spectrum antibiotics should be administered early and, ultimately, urgent surgery is required to close the peritoneum and irrigate the peritoneal cavity. The patient in this case became hypotensive and developed tachycardia in the ED after the completion of the abdominal and pelvic CT scans, despite aggressive fluid resuscitation. The patient developed a fever and, despite the administration of further rounds of parenteral pain medications, his pain worsened. No specific etiology for the perforation could be identified on the CT scan. The consultant surgeon brought the patient to the operating room and a large amount of bilious material was found within the abdomen during exploration, in addition to an ulcer on the anterior surface of the antrum of the stomach, near the pylorus. The perforation was closed with sutures, and the omentum was brought up and tacked over the perforation. The abdomen was irrigated copiously and the skin was then closed. The patient had an uncomplicated postoperative course on IV antibiotics, and he was discharged home after adequate recovery.

ww w. me

dic

alm cq sd ow

nlo

ad .bl og

sp ot. c

om

3o clinical cases from eMedicine Case 7

Background A 62-year-old man presents to the hospital with profuse diaphoresis and vomiting.

Figure 1

The pa

ent also complains of chest pressure. His blood pressure is 80 mm Hg systolic, 50 mm Hg diastolic.

ww w. me

dic

alm cq sd ow

Hint

nlo

ad .bl og

sp ot. c

om

DIAPHORESIS AND VOMITING IN A 62-YEAR-OLD MAN

3o clinical cases from eMedicine Case 7 Answer

Diagnosis: Inferior-wall acute myocardial infarction Discussion:

Note the ST-segment elevation in leads II, III, and aVF, as well as reciprocal changes in leads I and aVL. STsegment elevations also are present in lateral leads. Findings in the right-sided leads were consistent with right ventricular infarction; this finding was confirmed during cardiac catheterization.

ww w. me

dic

alm cq sd ow

nlo

ad .bl og

sp ot. c

om

DIAPHORESIS AND VOMITING IN A 62-Y EAR -O LD MAN

3o clinical cases from eMedicine o Case 8

Background

ww w. me

dic

On physical examina on, the pa ent has an elevated temperature of 101.3F (38.5C), a blood pressure of 130/76 mm Hg, a pulse of 110 bpm, and a respiratory rate of 20 breaths/min. The pa ent is not in acute distress, but he is mildly ill-appearing and diaphoretic. His oropharynx is clear, with slightly dry mucous appearing diaphoretic. membranes. His lungs are clear to auscultation, and his heart rate is regular, without murmurs. The abdominal

alm cq sd ow

Figure 1

nlo

ad .bl og

A 60-year-old man presents to the emergency department (ED) complaining of a 1 old 1-day history of crampy, moderately intense, left-sided abdominal pain. The pain is constant in nature and exacerbated by movement; it sided is relieved by lying still. The patient has not experienced anorexia and has not eaten since the evening before. He has had several loose brown stools but denies any nausea or vomiting. The stool in his bowel movements is not blood streaked and does not appear tarry. He denies any recent travel or camping and has not eaten any uncooked or undercooked foods. He reports feeling febrile, sweaty, and generally fatigued. No urinary oked symptoms, such as dysuria or increased frequency, are reported. He has not had any recent contact with sick people. He denies having had similar episodes in the past. His medical and surgical histories are unremarkable, the although he did have a screening barium enema examina on 3 years ago. He is a nonsmoker and denies any heavy or regular alcohol consumption. He does not take any prescription or over the-counter medications. over-the

sp ot. c

om

LEFT LOWER QUADRANT ABDOMINAL PAIN IN A 60-YEAR-OLD MAN

3o clinical cases from eMedicine


examination reveals moderate tenderness in the left lower quadrant, with voluntary guarding. There is no rebound tenderness. No costovertebral angle tenderness or inguinal hernias are appreciated, and his genital exam is noted to be normal. On digital rectal examination, the patient is tender on the left side of the rectal vault, and the stool is noted to guaiac-negative. The remainder of the physical examination is unremarkable. Serum laboratory tes ng is remarkable only for an elevated white blood cell (WBC) count of 16.0 103/L (16.0 109/L), with a neutrophil predominance; the urinalysis is unremarkable. A standard radiograph of the abdomen is obtained, which does not show any signicant abnormali es (see Figure 1). The pa ent then undergoes a computed tomography (CT) scan of the abdomen and pelvis (see Figures 2 and 3).

ww w. me

dic

alm cq sd ow

Figure 2

nlo

ad .bl og

sp ot. c

om

3o clinical cases from eMedicine

What is the diagnosis?

Hint: This is the most common acute condition related to the sigmoid colon. o o o o Acute diverticulitis Colon cancer Acute bacterial peritonitis Acute appendicitis

ww w. me

dic

alm cq sd ow

Figure 3

nlo

ad .bl og

sp ot. c

om

3o clinical cases from eMedicine o Case 8 Answer

Discussion:

ww w. me

dic

alm cq sd ow

Acute diverticulitis results from inflammation of a diverticulum (small mucosal and submucosal herniations through the circular muscle layer of the colonic wall) secondary to fecal obstruction. The obstruction typically obstruction. occurs at the neck of the diverticulum; solidified stool, which typically forms a fecalith, abrades the mucosa within or at the neck of the diverticulum. In uncomplicated cases (typically characterized by a well well-appearing patient without peritonitis and systemic signs/symptoms), the inflammatory process is confined to the colonic atient wall; however, the obstruction, with subsequent high intraluminal pressure within the diverticula, can lead to a microperforation which, in turn, allows translocation of bacteria through the colonic wall, pericolic abscess llows forma on, and diuse peritoni s. Only 4% of pa ents diagnosed with diver culi s are younger than 40 years -4% 2 old; the condition is predominantly found in elderly populations.

Figure 1

nlo

ad .bl og

Diagnosis: Acute diverticulitis

sp ot. c

om

LEFT LOWER QUADRANT ABDOMINAL PAIN IN A 60-YEAR -OLD MAN

3o clinical cases from eMedicine

ww w. me

dic

alm cq sd ow

Figure 2

Figure 3

nlo

ad .bl og

sp ot. c

om

3o clinical cases from eMedicine


The colonic diverticula themselves are most commonly found in the sigmoid and descending colon, although, less commonly, patients develop diverticula of the right colon (particularly in patients of Asian descent). The condition of diverticulosis is an intestinal disorder that is characterized by the presence of many diverticula and which occurs equally in men and women, with a higher prevalence in cultures with a low-fiber diet (a low-fiber diet is believed to decrease stool transit time, thereby causing increased intraluminal pressure and resulting in mucosal hernia ons). Approximately one third of the popula on has diver culosis by age 50 years, and about two thirds have it by age 85 years. Approximately 10-25% of pa ents with known diver culosis go on to develop diverticulitis. The classic presentation of diverticulitis consists of steady, deep abdominal pain that is often initially diffuse and vague, but later localizes in the left lower quadrant of the abdomen. Abdominal bloating, stool changes such as diarrhea or constipation, and flatulence frequently accompany acute diverticulitis. Fevers, fatigue, and anorexia are also common complaints. Colonic inflammation may irritate the bladder or the ureters, leading to complaints of urinary frequency and dysuria. A physical examination may reveal fever; localized, left lower quadrant abdominal tenderness; mild abdominal distention; and, at times, a left lower quadrant mass. The palpated mass is likely to be inflamed loops of bowel or, possibly, an abscess. A digital rectal examination may demonstrate left-sided tenderness and occult blood in the stool. The differential diagnosis of acute sigmoid diverticulitis is broad and includes inflammatory bowel disease, irritable bowel syndrome, appendicitis, ischemic colitis, colon cancer, urolithiasis, urinary tract infection, and, in women, a number of obstetric/gynecologic conditions (such as tubo-ovarian abscesses and ovarian cysts). The complications of acute diverticulitis include the formation of a pericolic abscess, frank colonic perforation leading to free intra-abdominal air, local adhesions, purulent or fecal peritonitis, sepsis, bowel obstruction, and fistula formation between the colon and the bladder or vagina. Fistula formation is more common in the setting of recurrent diverticulitis, with the most common type being a colovesicular fistula that is characterized by fecaluria, pneumaturia, or typical urinary tract infection symptoms. The initial evaluation of a patient with suspected acute diverticulitis generally includes a physical examination, complete blood cell count, urinalysis, and, when indicated by the presence of peritonitis, plain x-rays of the abdomen to rule out colonic perforation. Plain films are of limited value; however, they may show colonic obstruc on, mild ileus, or bowel disten on. Leukocytosis is found in only 36% of cases of acute diver culi The preferred imaging modality for the diagnosis of acute diverticulitis is CT scanning, as it detects both the extent of the disease and the presence of complications. Abdominal ultrasonography can also be used, but it lacks specificity and is operator-dependent. Barium contrast studies and colonoscopy/sigmoidoscopy should be avoided in the setting of acute diverticulitis because of the risk of bowel perforation; however, these examinations are often performed after resolution of the acute stage in order to evaluate for the presence of complications, such as fistula formation or other colonic abnormalities. The management of patients with acute diverticulitis depends upon the severity of the illness, but it is most commonly successful with medical management alone. Well-appearing patients who are able to tolerate oral intake and who do not have systemic symptoms, peritonitis, or complications seen on CT scans may be treated as outpatients. In fact, reliable, nontoxic-appearing patients with a history of diverticulitis who present with their typical symptoms may even be treated empirically as outpatients, without repeat imaging, if no significant comorbidities (eg, an immunocompromised state, diabetes, or malignancy) exist. All patients treated at home require close follow-up care and reexamination, and they should be given detailed return precautions for worsening pain or systemic illness. Treatment of uncomplicated acute diverticulitis consists of bowel rest, broad-spectrum antibiotics, and pain control. Outpatients may be instructed to begin with a clear liquid diet and advance slowly as tolerated, whereas inpatients should be kept hydrated with intravenous fluids. Antibiotic regimens should cover gram-negative bacteria and anaerobes. A combination of either trimethoprimsulfamethoxazole or ciprofloxacin, with either metronidazole or clindamycin, is the primary recommended treatment regimens. Monotherapy with amoxicillin/clavulanic acid is an acceptable alternative regimen. Patients should be admitted to the hospital if they cannot tolerate oral intake of fluids, are immunocompromised, demonstrate signs of systemic toxicity (such as tachycardia and fever), or have developed evidence of peritonitis or intra-abdominal complications. These patients should receive nothing by

alm cq sd ow

nlo

ad .bl og

sp ot. c

om

s.

ww w. me

dic

3o clinical cases from eMedicine


mouth (NPO) and should be given intravenous antibiotics. Ciprofloxacin or an aminoglycoside may be paired with metronidazole or clindamycin as the recommended antibiotic regimen. A monotherapeutic agent, such as piperacillin/tazobactam, ampicillin/sulbactam, or ertapenem, may also be used. Selected abscesses detected by ultrasonography or abdominal CT scanning may be drained percutaneously, whereas perforations, fecal peritonitis, and fistula formation all require a surgical consultation. Abscesses less than 5 cm in diameter can be treated with an bio cs alone, although eval uation by a surgeon should still be sought. Recurrent diverticulitis and complicated diverticulitis are indications for partial colonic resection. Approximately 10-25% of pa ents who are medically managed have recurrent a acks and are at an increased risk of subsequent complica on. Interes ngly, pa ents younger than age 40 years are more likely to suer from recurrences and are more likely to benefit from elective sigmoid resection. In this case, the axial CT scan images of the abdomen at the level of the pelvis (see Figures 2 and 3) show acute diver culi s of the sigmoid colon, with mul ple diver cula (arrow heads), wall thickening (arrow in Figure 3), and inammatory stranding in the sigmoid mesentery (asterisk in Figure 2). There is no free air or abscess forma on. The screening barium enema performed 3 years ago (Figure 4) shows mul ple diver cula in the sigmoid and descending colon (arrowheads). As a result of systemic signs and symptoms of infection, this patient was admitted to the hospital. He was placed on bowel rest and started on intravenous metronidazole and ciprooxacin. Over the next 2 days, the pa ent defervesced and his leukocytosis resolved. His diet was advanced to a full diet, and he was discharged from the hospital on a 10-day course of amoxicillin/clavulanic acid.

ww w. me

dic

alm cq sd ow

nlo

ad .bl og

sp ot. c

om

3o clinical cases from eMedicine Case 9

Background

ww w. me

dic

alm cq sd ow

Figure 1

nlo

ad .bl og

A 14-year-old boy presents to the emergency department (ED) with a 10-day history of progressive weakness. The patient reports experiencing rhinorrhea, cough, and malaise approximately 3 weeks before admission. He developed lower-extremity weakness and diculty walking 8 days a er the onset of the upper respiratory tract infection symptoms. He was evaluated at a local hospital, where he was diagnosed with dehydration, treated with intravenous fluids, and discharged to home. Despite these measures, his lower-extremity weakness did not improve. Over the following 7 days, he began experiencing diuse muscle pain and progressive weakness that extended to his upper extremi es. During the 3 days before this presenta on, he developed a hoarse voice and shortness of breath. He also notes that he is now having difficulty urinating and has decreased oral intake. He currently denies having any fever, cough, vomiting, or diarrhea. The patient's past medical history is significant only for attention deficit hyperactivity disorder (ADHD), for which he takes methylphenidate. He has had no previous hospitalizations, has no known drug allergies, and has had all recommended childhood immunizations. His family history is noncontributory.

sp ot. c

om

A 14-YEAR-OLD BOY WITH PROGRESSIVE WEAKNESS AND DYSPNEA

3o clinical cases from eMedicine


The physical examination reveals an afebrile, ill-appearing teenager, with a heart rate of 118 bpm, a respiratory rate of 28 breaths/min, a blood pressure of 168/122 mm Hg, and an oxygen satura on of 93% while breathing room air. Auscultation of the lungs reveals diffuse, poor aeration. His heart sounds are normal, without any appreciable murmur. His strength is symmetric but diminished to 2/5 in his lower extremi es and 4/5 in his upper extremi es (5/5 being normal strength). The pa ent's sensa on is intact to light touch, but there is a loss of vibratory sense. He has no deep tendon reflexes in his lower extremities, diminished deep tendon reflexes (1+) in his upper extremi es, and absent plantar reexes. Cranial nerves II are intact; however, he has a -XII weak cough and gag reflex, with impaired handling of secretions. The remainder of his examination is unremarkable. The patient is intubated for progressive respiratory distress and loss of airway-protective reflexes. He is fluidresuscitated with a liter of intravenous normal saline. An electrocardiogram (ECG) is obtained, which demonstrates sinus tachycardia. The initial laboratory analysis, including a complete blood cell (CBC) count and a basic metabolic panel, is within normal limits. A lumbar puncture is performed, with an opening pressure of 15 cm H20. The cell count and Gram stain of the cerebrospinal uid (CSF) demonstrates 2 white blood cells per high power eld, 4 red blood cells per high power eld, and no organisms. Addi onal analysis of the CSF shows a protein concentra on of 96 mg/dL (960 mg/L) and glucose concentra on of 72 mg/dL (3.99 mmol/L). The patient is sent for magne c resonance imaging (MRI) of his brain and spine (see Figure 1) and is transported to the pediatric intensive care unit (ICU) for further management.

What is the patient's condition as verified by the MRI?

Hint: Look closely at the cauda equina on the MRI images. o o o o Spinal epidural abscess Guillain-Barr syndrome Transverse myelitis Multiple sclerosis

ww w. me

dic

alm cq sd ow

nlo

ad .bl og

sp ot. c

om

3o clinical cases from eMedicine o Case 9 Answer

Diagnosis: Guillain-Barr syndrome Barr Discussion:

ww w. me

dic

alm cq sd ow

The lumbrosacral MRIs (see Figures 1 and 2) demonstrate nerve root enhancement of the cauda equina on axial post-contrast T1-weighted sequences. The localization of progressive weakness includes spinal cord lesions weighted (such as transverse myelitis or anterior spinal artery syndrome), peripheral neuropathies (such as those caused peripheral by heavy metals), neuromuscular junction diseases (such as that caused by organophosphate pesticides), myasthenia gravis, botulism, and myopathies (such as dermatomyositis). The presence of progressive ascending weakness, areflexia, autonomic dysfunction, elevated CSF protein without pleocytosis, and enhancement of the cauda equina nerve roots on lumbrosacral MRIs make the diagnosis of Guillain Barr syndrome most probable Guillain-Barr in this patient.

Figure 1

nlo

ad .bl og

sp ot. c

om

A 14-YEAR -OLD BOY WITH P ROGRESSIVE WEAKNESS AND DYSPNEA

3o clinical cases from eMedicine

The typical presentation of Guillain-Barr syndrome is fine paresthesias in the toes and fingertips, followed by symmetric lower-extremity weakness that may ascend, over hours to days, to involve the arms and the muscles of respiration. Pain, predominately back, lower-limb and abdominal pain, is often a prominent feature of the syndrome. The physical examination reveals symmetric weakness, with diminished or absent reflexes and variable loss of sensation in a stocking-glove distribu on. Signs of autonomic dysfunc on are present in 50% of patients, and they include cardiac dysrhythmias, orthostatic hypotension, transient or persistent hypertension, ileus, constipation, and bladder dysfunction. Deviation from the classic presentation of ascending progression of

ww w. me

Guillain-Barr syndrome is an acute, idiopathic, monophasic, acquired inflammatory demyelinating polyradiculoneuropathy (AIDP) that affects both children and adults. It is a heterogeneous syndrome, with several variant forms. AIDP is the prototype of Guillain-Barr syndrome, and it is the most common form in North America, Europe, and most of the developed world (where it accounts for about 85-90% of cases). Guillain-Barr syndrome can occur at any age, but there appears to be a bimodal distribution, with peaks in young adulthood (15-35 y) and in the elderly (50-75 y). The cause of Guillain-Barr syndrome is unknown, but the disorder is thought to result from a postinfectious immune-mediated process called molecular mimicry that predominantly damages the myelin sheath of peripheral nerves. Approximately two thirds of patients report a history of an antecedent respiratory tract or gastrointes nal infec on 2 weeks before the onset of neurologic -4 symptoms. A variety of infectious agents have been associated with Guillain-Barr syndrome, although Campylobacter is the most frequent. Other organisms that commonly precede Guillain-Barr syndrome include cytomegalovirus, Epstein-Barr virus, Haemophilus influenzae, Mycoplasma pneumoniae, the enterovirus family, hepatitis A and B, herpes simplex virus, and Chlamydophila (formerly Chlamydia) pneumoniae.

dic

alm cq sd ow

Figure 2

nlo

ad .bl og

sp ot. c

om

3o clinical cases from eMedicine


weakness is not uncommon. In what is known as the Miller-Fisher variant, cranial nerves are aected in 30-40% of patients at any time in the course of the syndrome. This form of the disease is also characterized by areflexia, ataxia and ophthalmoplegia. The facial nerves are most commonly involved, resulting in bilateral facial weakness. Although the associated autonomic dysfunction may produce life-threatening complications, mortality from Guillain-Barr syndrome is largely secondary to respiratory failure associated with respiratory muscle weakness. Approximately 20% of children with Guillain-Barr syndrome require mechanical ventilation for respiratory failure. The need for intubation should be anticipated early so that it can be done nonemergently in a controlled environment. Progression to respiratory failure has been predicted in patients with rapid disease progression, bulbar dysfunction, bilateral facial weakness, or dysautonomia. Emergent intubation should be performed in any patient with loss of the gag reflex, declining respiratory function, or pharyngeal dysfunction. Care should be taken during intubation, as autonomic dysfunction may complicate the use of vasoactive and sedative drugs. A er the rst week of symptoms, analysis of the CSF typically reveals normal opening pressures, fewer than 10 white blood cells per high power field (typically mononuclear), and an elevated protein concentration (greater than 45 mg/dL). This nding, also known as albuminocytologic dissocia on, may be delayed. As a result, a repeat lumbar puncture may be required as the protein values may not rise for 1-2 weeks, and maximum protein values may not be seen for 4-5 weeks. In addi on, gadolinium -enhanced lumbosacral MRI may demonstrate enhancement of the cauda equina nerve roots. This imaging modality has been described to be 83% sensi ve for acute GuillainBarr syndrome, and abnormalities are present in 95% of typical cases. Electrophysiologic studies are the most specific and sensitive tests for confirming the diagnosis. Most patients demonstrate slowing of nerve conduc on 23 weeks a er the onset of symptoms. There are a variety of abnormalities seen in Guillain-Barr syndrome that indicate evolving multifocal axonal demyelination in peripheral nerves, spinal roots and/or cranial nerves. Abnormalities seen on electromyography include partial motor conduction block, slowed nerve conduction velocities, abnormal temporal dispersion, and prolonged distal latencies. The earliest finding, which may be present within days of symptom onset, is prolongation or absence of the F responses, which indicates demyelination involving the proximal nerve roots. Any patient presenting with a clinical picture consistent with Guillain-Barr syndrome requires immediate hospitalization. The indications for admission to the ICU include, but are not limited to, respiratory insufficiency or failure, loss of airway-protective reflexes, and severe autonomic instability. The main modalities of therapy for Guillain-Barr syndrome include plasma exchange and intravenously administered immunoglobulin (IVIG). Corticosteroids have not been shown to be beneficial. The American Academy of Neurology issued a practice parameter regarding immunotherapy for Guillain-Barr syndrome that concluded IVIG and plasma exchange are options for children with severe disease and should be reserved for those with the following findings: Rapidly progressing weakness

Significant bulbar weakness Inability to walk unaided

Several trials have demonstrated that IVIG is at least as effective as plasma exchange in the treatment of Guillain-Barr syndrome and is associated with a lower rate of complica ons. IVIG administered at 0.4 g/kg/day for 5 days has been shown to hasten recovery and lower the relapse rate. Doses of 1 g/kg/day over 2 days have also been demonstrated to hasten recovery time, but early relapses are more prevalent. The combination of plasma exchange and IVIG does not improve outcomes or shorten the duration of illness. Complications associated with Guillain-Barr syndrome include arrhythmia, sepsis, pneumonia, ileus, deep venous thrombosis and pulmonary embolism. The risk of sepsis and infection may be decreased by aggressive

ww w. me

Worsening respiratory status or need for mechanical ventilation

dic

alm cq sd ow

nlo

ad .bl og

sp ot. c

om

3o clinical cases from eMedicine


physiotherapy and mechanical ventilation with positive end expiratory pressure (PEEP). Administration of anticoagulant therapy and intermittent pneumatic compression devices may lower the risk of deep venous thrombosis and pulmonary embolism. Cardiac telemetry is useful to monitor for arrhythmias, which are a common cause of morbidity and mortality in Guillain-Barr syndrome. In addition, physical and occupational therapy should be initiated early and may be beneficial in helping patients to regain their baseline functional status. More than 90% of pa ents reach the nadir of their func on within 4 weeks of the onset of symptoms, with return of normal function occurring slowly over the course of weeks to months. The majority of patients with Guillain-Barr syndrome achieve a full and func onal recovery within 612 months. The clinical course of Guillain-Barr syndrome in children is shorter than it is in adults, and recovery is more complete. Upon transferring this patient to the ICU, a dialysis catheter was placed and plasmapheresis was initiated. His hypertension was controlled with a nicardipine drip. Prophylaxis for deep venous thrombosis was started. On hospital day 3, the pa ent had improved strength in his lower extremi es and pressure support trials on the ven lator were ini ated. On hospital day 6, he was extubated successfully to room air a er receiving a total of 5 sessions of plasmapheresis. He was transferred to a pediatric ward 1 week a er admission with intensive physical and occupational therapies. At the time of the transfer, the patient's bulbar symptoms were resolved, and his strength was 3/5 in his lower extremi es and 4/5 in his upper extremi es.

ww w. me

dic

alm cq sd ow

nlo

ad .bl og

sp ot. c

om

3o clinical cases from eMedicine Case 10

Background

ww w. me

dic

What is the diagnosis?

Hint: The ECG results suggest a chronic condition. o Pulmonary embolus o Chronic obstructive pulmonary disease o Acute bacterial pneumonia o Stable angina

alm cq sd ow

Figure 1

nlo

On physical examination, the patient is alert but appears to be in mild respiratory distress, with moderate retractions and pursed-lipped breathing. He is afebrile. His blood pressure is 140/85 mm Hg, and his pulse rate is 103 bpm and mostly regular. His respiratory rate is 28 breaths/min, and a pulse oximetry reading shows 85% while the patient is breathing room air. His breath sounds are diminished throughout, with a markedly prolonged expiratory phase and faint expiratory wheezes in the upper lung fields. The cardiac examination reveals distant heart sounds with a somewhat prominent P2. He has no murmur, gallop, or pericardial rub. His skin is cool and dry. He has trace edema at his ankles but no cyanosis or clubbing. An electrocardiogram (ECG) is performed (see Figure 1).

ad .bl og

He describes worsening dyspnea on exertion that is associated with chest tightness, wheezing, and coughing. The patient's dyspnea has worsened to the point that he can hardly walk from his couch to the bathroom without becoming extremely short of breath. He recently recovered from a cold, with several days of nasal congestion, clear rhinorrhea, and a nonproductive cough. He reports having been healthy his whole life and has not been to see a physician in at least 2 decades; however, he does admit that he has gradually curtailed his physical activities, such as gardening, shoveling snow, and walking in the mall, because he has been increasingly "ge ng winded." He smokes 2 packs of cigare es daily, a habit he has been trying to break for at least 30 years .

sp ot. c

A 65-year-old man presents to the emergency department (ED) complaining of difficulty breathing.

om

AN ELDERLY MAN WITH DYSPNEA

3o clinical cases from eMedicine o Case 10 Answer

Diagnosis: Chronic obstructive pulmonary disease Discussion:

The ECG demonstrates a constellation of findings that suggest COPD, including sinus tachycardia, a rightward axis, P pulmonale (a P wave amplitude > 2.5mV in the inferior leads), a low QRS voltage, and a right bundle branch block (RBBB). In addition, the slight ST segment depressions in the inferior leads are suggestive of prominent atrial depressions repolarization abnormalities and are seen in COPD COPD.

ww w. me

dic

Chronic obstructive pulmonary disease (COPD) is currently the fourth leading cause of death in the United States. COPD is defined as a disease state characterized by airflow obstruction caused by chronic bronchitis or emphysema. The airflow obstruction generally is progressive, and it may be accompanied by partially reversible airway hyperreactivity. The condition was rst described in Western Europe in the early 19th century by Badham (1808) and Laennec (1827), who made the classic descrip on of chronic bronchi s and emphysema. A Bri sh medical textbook of the 1860s described the familiar clinical picture of chron bronchitis as an advanced ic disease, with repeated bronchial infections, that ended in right heart failure. The modern definition of chronic bronchitis and emphysema which incorporated the concept of airflow obstruction was proposed by participants of the Ciba symposium of 1958.

alm cq sd ow

Figure 1

nlo

ad .bl og

sp ot. c

om

AN ELDERLY MAN WITH DYSPNEA

3o clinical cases from eMedicine


Chronic bronchi s is dened, in clinical terms, as the presence of a chronic produc ve cough for 2 consecu ve years, las ng for at least 3 months during each year, at the exclusion of other e ologies. Emphysema is dened, in terms of anatomic pathology, as an abnormal, permanent enlargement of the air spaces distal to the terminal bronchioles, accompanied by destruction of their walls and without obvious fibrosis. The characteristic histopathology of chronic bronchitis is mucous gland hyperplasia with bronchial wall thickening, focal squamous metaplasia, ciliary abnormalities, and variable amounts of airway smooth muscle hyperplasia and inflammation. These changes lead to luminal occlusion causing airflow limitation by allowing airway walls to deform and narrow the airway lumen. Emphysema has 3 morphologic pa erns: centriacinar emphysema (focal destruc on limited to the respiratory bronchioles and the central portions of acinus; associated with cigarette smoking), panacinar emphysema (involves the entire alveolus distal to the terminal bronchiole; generally develops in pa ents with homozygous alpha1antitrypsin [AAT] deficiency), and distal acinar emphysema or paraseptal emphysema (the least common form, involving distal airway structures, alveolar ducts, and sacs; it is localized to fibrous septa or to the pleura and leads to formation of bullae leading to pneumothorax). In the United States, approximately 14.2 million people have COPD; approximately 12.5 million of these cases stem from chronic bronchi s and the remaining 1.7 million stem from emphysema. Since 1982, the number of pa ents diagnosed with COPD increased by 41.5%. The prevalence of chronic airow obstruc on in the United States is es mated at 817% for men and 10-19% for women. Interna onally, the rates likely are higher because more than 1.2 billion humans are exposed to the ravages of smoking. Absolute mortality rates for pa ents in the United States aged 55-84 years (1985) were 200 per 100,000 males and 80 per 100,000 females. Internationally, a marked variation in overall mortality rates from COPD exists. The extremes are the more than 400 deaths per 100,000 males aged 65-74 years in Romania and the fewer than 100 deaths per 100,000 in Japan. Patients with COPD are susceptible to many conditions that can rapidly lead to an acute deterioration superimposed on chronic disease. Quick and accurate recognition of these patients, along with aggressive and prompt intervention, may be the only action that prevents frank respiratory failure. The disease is not generally diagnosed on the basis of ECG findings; however, the signs and symptoms of cardiac and pulmonary disease may overlap substantially, and ancillary testing can be useful in establishing the diagnosis. In fact, it may not be unusual for ECG to be the first diagnostic test performed in patients with longstanding COPD, if patients present with shortness of breath as part of a general workup for a possible cardiac etiology for the symptoms. Knowledge of the usual ECG manifestations of COPD enables the clinician to recognize uncharacteristic abnormalities, which often represent the effects of superimposed illnesses or drug toxicity. A tachycardic rhythm is common in individuals who are experiencing exacerbations of their COPD as a compensatory mechanism for hypoxia or poor right ventricular function (in the setting of cor pulmonale). Sinus tachycardia is the most common form reported in the literature, but other supraventricular arrhythmias, such as atrial tachycardia (unifocal or multifocal), atrial fibrillation, and atrial flutter, can also be present. P pulmonale (ie, a P wave amplitude >2.5 mm) is frequently reported but is a rela vely insensi ve predictor of right atrial enlargement. In patients with COPD, the amplitude of the P wave is in fact dynamic, and it tends to be more prominent during acute exacerbation than at other times. A vertical or rightward axis is another manifestation of pulmonary hypertension. Similarly, complete or incomplete RBBB, right ventricular hypertrophy, or both commonly occur in patients with cor pulmonale. Low voltage, particularly in the limb leads, is another ECG characteristic of patients with COPD. This finding is classically attributed to increased impedance through a hyperinflated chest; however, low voltage is not directly correlated with hyperinflation, and it is neither sensitive nor specific for COPD. The mainstays of therapy for acute exacerbations of COPD, as in this case, are oxygen, bronchodilators, and steroids. Adequate oxygen should be given to relieve the hypoxia. Supplemental oxygen should maintain a nearnormal satura on to above 90%. Bronchodilator therapy with both betaagonist and anticholinergic nebulizer therapy should be administered promptly. The need for intubation should be established and performed if indicated (e.g., hypoxia not relieved with supplemental oxygen, severe respiratory distress, obtundation). If

ww w. me

dic

alm cq sd ow

nlo

ad .bl og

sp ot. c

om

3o clinical cases from eMedicine


necessary and available, continuous positive airway pressure (CPAP) may be used. Other therapies that are indicated in specific situations may include antibiotics, magnesium, and Heliox (a mixture of helium and oxygen that leads to increased oxygen delivery as a result of improved laminar flow). The patient in this case was given a combination beta-agonist and anticholinergic nebulizer therapy for his respiratory distress and hypoxia. A chest radiograph was performed, which revealed a moderately sized basilar infiltrate. Antibiotics appropriate for community-acquired pneumonia coverage and intravenous steroids were administered, and the patient was admitted to the hospital for continued inpatient therapy and monitoring. On the morning of hospital day 3, the pa ent was noted to be breathing easier and was discharged to home with prescriptions for maintenance inhaled steroid therapy, and for beta-agonist and anticholinergic metered-dose inhalers (MDIs). Appropriate follow-up was arranged for continued outpatient management.

ww w. me

dic

alm cq sd ow

nlo

ad .bl og

sp ot. c

om

3o clinical cases from eMedicine Case 11

Background

Figure 1

Hint

You could hit a golf ball off of those T waves.

ww w. me

dic

alm cq sd ow

nlo

ad .bl og

sp ot. c

A 55-year-old woman with type 2 diabetes mellitus states that she has been feeling "weak all over for a few days." She has generalized edema, including edema of the hands, legs, and eyelids.

om

WEAK ALL OVER

3o clinical cases from eMedicine Case 11 Answer

Diagnosis: Hyperkalemia Discussion:

In the setting of hyperkalemia, the T waves may start to become increasingly tall and peaked at a serum potassium level of 5.5 mmol/L. The QRS complex may begin to widen at a serum potassium level of 6.5 mmol/L. The P wave may begin to flatten at a serum potassium level of 7 mmol/L. Note the PR prolonga on. PR prolongation caused by hyperkalemia is usually primarily due to an increase in the P wave duration. This patient has a first-degree atrioventricular (AV) block. At a serum potassium level of 8 mmol/L, the P wave may become invisible. This pa ent had a serum potassium level of 7.1 mmol/L and a pH of 7 in the se ng of renal failure. Her generalized edema was the result of a 25-lb weight gain due to the renal failure.

ww w. me

dic

alm cq sd ow

nlo

ad .bl og

sp ot. c

om

WEAK ALL OVER

3o clinical cases from eMedicine Case 12

Background

ww w. me

dic

alm cq sd ow

Figure 1

nlo

On arrival to the hospital, the patient is ill-appearing and comba ve. His ini al vital signs are a heart rate of 117 bpm, a blood pressure of 85/50 mm Hg, a respiratory rate of 32 breaths/min, and an oxygen satura on of 91% on the non-rebreather mask. On primary survey, his oropharynx is clear, his airway is patent, and his trachea appears to be shifted to the right of midline. On auscultation, the patient's breath sounds are decreased over the left chest. Percussion of the left chest demonstrates hyperresonance. His carotid pulse is weakly palpable, and his jugular venous pulse is elevated. The pa ent receives a Glasgow Coma Scale score of 12. The pa ent's clothing is removed, revealing no obvious deformities or areas of bleeding. The patient's abdomen is soft, without any tenderness to palpation. His pelvis is stable. Standard trauma x-rays, including an anteroposterior (AP) chest and pelvis scan, are performed after the primary survey. A complete secondary survey is postponed because of the patient's poor clinical condition.

ad .bl og

sp ot. c

A 26-year-old man with an unknown past medical history arrives to the emergency department (ED) by ambulance. He had been driving his car while unrestrained and was involved in a high-speed motor vehicle collision. There was airbag deployment and significant front-end damage to the vehicle, with intrusion into the passenger compartment of the car. The patient was extricated from the vehicle and placed on a backboard, and a cervical collar was placed by EMS. A non-rebreather facemask and 1 peripheral intravenous (IV) line were placed in the field.

om

THE UNRESTRAINED DRIVER

3o clinical cases from eMedicine

What is the underlying pathophysiology, and what procedure was performed?

Hint: The cause of this patient's hypotension and hypoxia is a clinical diagnosis, and although a portable chest radiograph was performed in this case, this condition should not typically require imaging. o o o o Upper airway obstruction; cricothyrotomy Tension pneumothorax; needle thoracostomy Hypovolumic shock; central line placement Pericardial tamponade; pericardiocentesis

ww w. me

dic

alm cq sd ow

nlo

ad .bl og

sp ot. c

A second large-bore peripheral intravenous line is placed, and the pa ent begins to receive a bolus of 1000 cc of normal saline under pressure. A decision to perform an emergent procedure is made. Immediately after the procedure is performed, the patient is noted to have a dramatic clinical improvement. Subsequent to the procedure, the pa ent has a pulse of 105 bpm, a blood pressure of 95/60 mm Hg, a respiratory rate of 22 breaths/min, and an oxygen satura on of 98% on the nonrebreather mask. The secondary survey is completed, revealing no major injuries. Addi onally, the chest radiograph (see Figure 1) conrms the suspected clinical diagnosis that prompted the emergent procedure.

om

3o clinical cases from eMedicine o Case 12 Answer

Discussion:

A pneumothorax in any patient who has sustained thoracic trauma should arouse suspicion. The patient may complain of an acute onset of sharp pleuritic chest pain, with radiation to the ipsilateral shoulder and pleuritic associated dyspnea and anxiety. Typical physical findings in pneumothorax include unilaterally decreased breath sounds, hyperresonance to percussion over the affected lung, and asymmetric ches rise. In tension chest pneumothorax, the patient displays respiratory distress, tachypnea, and tachycardia, and the patient may also experience cyanosis, jugular venous distention, tracheal deviation away from the affected lung, and a pulsus paradoxus. The epidemiology of traumatic pneumothoraces has not been well characterized. In the United States, trauma is demiology the leading cause of death in persons younger than 45 years, and it accounts for approximately 150,000 deaths annually. The overall mortality for thoracic trauma is 10%, and chest injuries cause approximately 1 in 4 trauma thoracic deaths in North America. Pneumothorax is a serious complication of thoracic trauma, and it has been described in 1 in 5 pa ents that survive major trauma. Interes ngly, in one study,2% of pa ents with asymptoma c 1 chest stab wounds had a delayed pneumothorax or hemothorax. While pneumothoraces in stable patients can be confirmed radiographically, a tension pneumothorax causing hemodynamic compromise should be diagnosed clinically, and treatment should never be delayed in favor of diagnostic imaging. A chest x-ray may show a linear shadow of visceral pleura, without lateral lung markings. An ray upright chest x-ray is more sensitive than a supine radiograph, as air tends to accumulate at the lung apex. In ray recumbent patients, air often accumulates in the anterior portion of the inf inferior chest and manifests radiographically as a "deep sulcus." If a pneumothorax without tension physiology is suspected but not seen on the initial upright chest x-ray, a repeat film during exhalation may reveal it. Increasingly, ultrasound is being ray, used as a rapid bedside modality for diagnosing pneumothoraces; some studies have shown that it is more sensitive than radiography for detecting traumatic pneumothoraces. Computed tomography (CT) is more sensitive and specific than chest x x-rays or ultrasonography for the evaluation of small pneumothoraces and hy hemothoraces. Occult pneumothoraces may be present in 2 55% of trauma pa ents, although the clinical 2-55%

ww w. me

dic

alm cq sd ow

Pneumothorax occurs when air enters the potential space between the visceral and parietal pleura, leading to lung collapse on the affected side. Pneumothoraces may occur spontaneously, especially in the setting of lung disease, or they may result from accidental or iatrogenic trauma. A tension pneumothorax is a life life-threatening condition that occurs when the air in the pleural space is under pressure, displacing mediastina structures and mediastinal compromising cardiopulmonary function. Tension pneumothoraces result from injuries to the lung parenchyma or bronchial tree that can act as one way valves so that air enters the pleural space but cannot escape. The one-way trapped air in a tension pneumothorax causes increased intrathoracic pressure, pushing mediastinal structures contralaterally and reducing venous return and cardiac output. These patients are hypoxic and become difficult to ventilate, with potentially rapid progress to cardiorespiratory collapse and death. Hemothorax is defined by cardiorespiratory blood in the pleural space, and it occurs when the lung parenchyma and the intercostal or mammary vessels are injured. Massive hemothoraces arise with hilar injuries, aortic ruptures, or myocardial ruptu ruptures. A tension hemopneumothorax develops when there is both blood and air under tension in the pleural space space.

nlo

ad .bl og

Diagnosis: Tension pneumothorax; needle thoracostomy

sp ot. c

om

THE UNRESTRAINED DRIVER

3o clinical cases from eMedicine


significance of occult pneumothoraces in patients who are not mechanically ventilated under positive pressure is unclear. Making the diagnosis of hemothorax may be more challenging. A minimum of 200-300 mL of blood is needed in the pleural space for blunting of the costophrenic angle to be visible on an upright chest x-ray. Blood is more difficult to appreciate on a supine x-ray because it will typically layer posteriorly, and ever larger volumes (up to 1000 mL) of blood may produce only a mild diuse radiodensity. Lateral chest lms may help differentiate hemothoraces from pulmonary contusions, and ultrasonography may also be useful for detecting fluid above the diaphragm. As with pneumothoraces, CT scanning is the most sensitive modality for diagnosing hemothoraces, although patients with massive hemothoraces may be too unstable for the scan. The treatment of traumatic pneumothoraces and hemothoraces depends upon the volume of blood or air that has accumulated and on the condition of the patient. Hemodynamically stable patients who are not intubated and have a rela vely small pneumothorax (ie, less than 1 wide) can be placed under observation. A repeat cm lm should be obtained a er 4 hours; if the pneumothorax is unchanged in size, the pa ent can con nue to -6 be observed without the need for decompression or tube thoracostomy. These patients should always be placed on 100% oxygen to increase the rate of reabsorp on of the air in the pleural space. In unstable pa ents who, on clinical grounds, are suspected of having a pneumothorax, a needle thoracostomy may be performed to quickly decompress the pleural space. A 14-gauge Angiocath (18-gauge or 20-gauge in an infant) should be placed immediately superior to the rib in the second intercostal space, midclavicular line on the affected side. Once in place, the needle is removed and the Angiocath is secured. A rush of air may be appreciated as the Angiocath enters the pleural space. Pneumothoraces should preferentially be decompressed either by needle decompression or placement of a tube thoracostomy before the patient is intubated, as positive pressure ventilation will exacerbate a pneumothorax; however, definitive management of the airway should never be delayed when indicated. Needle thoracostomy generally necessitates the subsequent placement of a chest tube; however, stable patients who do not require a chest tube may be observed. In simple spontaneous pneumothoraces, a 20F or 22F chest tube may be used; however, larger-caliber chest tubes (28F to 40F) should be used in most traumatic pneumothoraces and hemothoraces to ensure adequate drainage of any fluid. Chest tubes are placed in the fourth or fifth intercostal space in the anterior axillary or midaxillary line, and they should be directed posteriorly and toward the apex of the lung. After the tube is secured, it should be connected to a water seal and vacuum device, and placement should be confirmed by chest x-rays. In the case of a hemothorax, immediate drainage of more than 1500-2000 mL (or 20 mL/kg) of blood, or ongoing hemorrhage exceeding 600-1200 mL/6 hours (or >3 mL/kg/hr) a er the ini al drainage, constitute the definition of a massive hemothorax and generally are indications for a thoracotomy. Occasionally, placement of an additional chest tube may be necessary to assist in draining of the hemothorax. Additionally, the possibility of a bronchial injury should be considered if a continuing air leak is observed after several chest tubes and an unexpanded lung. In hemothorax, chest tubes should be directed posteriorly and inferiorly to arrive posterior to the diaphragm (as opposed to the placement for a simple pneumothorax). In this case, the junior emergency medicine resident placed a 14-gauge Angiocath in the second intercostal space, midclavicular line of the left chest. A rush of air was appreciated, and the patient's blood pressure (as previously noted in the case presenta on) improved to 95/60 mm Hg. The resident then prepared the le chest and placed a 38F chest tube in the h intercostal space, midaxillary line. There was immediate drainage of 1600 mL of bloody uid through the chest tube. Uncrossmatched blood was administered, and the surgical team was consulted for the massive hemothorax. The patient was intubated and transported to the operating room (OR). In the OR, the surgery team performed a thoracotomy, repaired the injured lung parenchyma, and ligated several small arteries that were actively bleeding. The patient was transported to the surgical intensive care unit (ICU) and extubated the following day. The chest tube was removed 48 hours later, and the pa ent was discharged on hospital day 4 in stable condi on .

ww w. me

dic

alm cq sd ow

nlo

ad .bl og

sp ot. c

om

3o clinical cases from eMedicine Case 13

Background

ww w. me

dic

What is the diagnosis? Hint

The medical history of this patient is significant.

alm cq sd ow

nlo

Figure 1

ad .bl og

A chest radiograph was obtained.

sp ot. c

A 47-year-old man presents with a chronic cough and chest discomfort that have lasted a few months. He does not have a fever, change of voice, or sputum production. He has no history of asthma and does not smoke. The clinical examination revealed generalized bronchial breathing. The findings were otherwise essentially normal. The medical history includes carcinoma of maxilla, which was treated with surgery and radiation therapy a few years ago.

om

CHRONIC COUGH

3o clinical cases from eMedicine Case 13 Answer

Diagnosis: Metastasis to the lungs (cannonball lesions). Discussion:

The chest radiograph depicts multiple opacities in both lung fields consistent with pulmonary metastases. These metastases are commonly known as cannonball lesions because of their appearance. Treatment includes referral to an oncologist and usually chemotherapy, depending on the primary malignancy.

ww w. me

dic

alm cq sd ow

nlo

ad .bl og

sp ot. c

om

CHRONIC COUGH

3o clinical cases from eMedicine Case 14

Background

dic

Hint

What is that straight line in the right lower lung field?

ww w. me

alm cq sd ow

nlo

Figure 1

ad .bl og

sp ot. c

A 45-year-old man complains of a fever, a cough productive of rust-colored sputum, and night sweats that have lasted 2 weeks. He is homeless and has alcoholism. What is the diagnosis?

om

FEVER, COUGH, AND NIGHT SWEATS

3o clinical cases from eMedicine Case 14 Answer

Diagnosis: Cavitary lesion, probable lung abscess Discussion:

ww w. me

dic

alm cq sd ow

nlo

ad .bl og

A circular lesion can be seen in the right lower lung field with the fluid level. This finding suggests the presence of a cavity filled with fluid. Lung abscesses are found in dependent regions of the lung such as the right lower lobe. The most common etiology is aspiration. If left untreated, this infection can extend into the plural space (ie, empyema). Approximately 90% of lung abscesses can be treated with an bio cs and do not require surgical evacuation. Antibiotics should include coverage for Bacteroides, Peptostreptococcus, and Fusobacterium species. First-line agents include clindamycin, cefoxitin, and piperacillin/tazobactam (Zosyn). If the symptoms fail to resolve, the patient should undergo an evaluation for a possible neoplasm.

sp ot. c

om

FEVER, COUGH, AND NIGHT SWEATS

3o clinical cases from eMedicine Case 15

Background

Two hours later, the nurse pages you because the pa ent's blood pressure had decreased to 75 mm Hg systolic, 50 mm Hg diastolic. Physical examina on reveals diuse and bilateral wheezing with dis nc ve heart sounds. A frontal chest radiograph (A) is obtained.

What should be performed immediately to reverse the hypotension?

Hint

Chest radiograph A reveals hyperinflated lung fields with the endotracheal (ET) tube properly positioned.

ww w. me

A. B. C. D.

Starting a dopamine infusion Placing a chest tube Decreasing the ventilator rate Increasing the tidal volume

dic

alm cq sd ow

Figure 1

nlo

ad .bl og

sp ot. c

A 67-year-old woman with a history of end-stage chronic obstructive pulmonary disease (COPD) is admitted to the hospital because of severe respiratory distress. She requires intubation in the emergency department because of respiratory acidosis. The patient's mechanical ventilator setting is in assist control (AC) mode, with the respiratory rate at 16 breaths per minute, the dal volume at 500 mL, and the frac on of inspired oxygen (FIO2) at 80%. Her vital signs are temperature, 100.4F (38C); heart rate, 104 beats per minute (bpm); pulse oximetry reading, 90%; and blood pressure, 130 mm Hg systolic, 80 mm Hg diastolic.

om

DOCTOR! THE PATIENT'S BLOOD PRESSURE IS FALLING!

3o clinical cases from eMedicine Case 15 Answer

Diagnosis: Hypotension secondary to severe respiratory distress Discussion:

Auto-PEEP frequently occurs in mechanically ventilated patients, especially those with asthma or COPD. It is the consequence of the ventilator delivering a positive-pressure breath before the patient has had time to completely exhale the previous breath. When a patient is intubated, a number of factors can contribute to the development of auto-PEEP: the ventilator rate, the end-inspiratory pause, and the inflation volume. Careful monitoring and repeated evaluation of high-risk patients are helpful to identify the problem. The expiratory flow waveform should be monitored, and if the flow does not return to baseline before the delivery of the next breath, auto-PEEP is present. If auto-PEEP is not recognized, inappropriate clinical decisions (in terms of diagnostics and interventions) may be made. A 2-minute trial of cessation of mechanical ventilation or decreasing the ventilator rate should be performed. The diagnosis is confirmed if the trial results in rapid improvement in hemodynamics. In general, auto-PEEP can be diminished by different means: decreasing the ventilatory frequency, decreasing the tidal volume, increasing the expiratory duration, sedating the patient, performing frequent suctioning, and using bronchodilators. Starting a dopamine infusion in this patient would not treat the cause of the hypotension, and placing a chest tube is indicated only in the case of a pneumothorax, which is one of the complications of auto-PEEP. During auto-PEEP, a portion of the tidal volume is trapped in the alveoli. Therefore, the answer is to decrease rather than increase the ventilator tidal volume. In this case, the flow and pressure waveforms revealed an auto-PEEP of 12 cm H2O. The respiratory rate was decreased to 10 breaths per minute, and another chest radiograph (B) was obtained shortly afterward. The pa ent^s blood pressure increased and stabilized at 130 mm Hg systolic/ 70 mm Hg diastolic.

ww w. me

dic

alm cq sd ow

nlo

ad .bl og

The correct answer is C, decreasing the ventilator rate. The cause of this patient's hypotension is the auto positive end-expiratory pressure (auto-PEEP) secondary to the hyperinflated lungs induced by a rapid ventilatory rate. Venous return is decreased because of elevated intrathoracic pressures, which cause a decrease in cardiac output and blood pressure. Auto-PEEP, intrinsic positive end-expiratory pressure (PEEP), occult PEEP, and positive end-expiratory alveolar pressure are terms that have been used to describe the failure of alveolar pressure to return to zero (atmospheric pressure) at the end of exhalation.

sp ot. c

om

DOCTOR! THE PATIENT'S BLOOD PRESSURE IS FALLING!

3o clinical cases from eMedicine Case 16

Background

ww w. me

dic

Hint

Signicant features of the history of this pa coronary artery bypass surgery.

alm cq sd ow

Figure 1

ent include an appendectomy 12 years ago, hypertension, and

nlo

ad .bl og

sp ot. c

A 76-year-old man presents with nausea, vomiting, diarrhea, and worsening abdominal pain. The pain is poorly localized and colicky and has lasted for 2 days.

om

ABDOMINAL PAIN, VOMITING, AND DIARRHEA

3o clinical cases from eMedicine Case 16 Answer

Diagnosis: Early small-bowel obstruction (SBO) Discussion:

ww w. me

dic

alm cq sd ow

nlo

ad .bl og

The radiograph was obtained with the patient in an upright position; therefore, it reveals multiple loops of small bowel filled with fluid. Straight lines of fluid can be seen in the left upper quadrant, and a single line of fluid can be seen in the right lower quadrant. No air is depicted in the colon or rectal region. The small bowel with the fluid is not yet dilated; this finding indicates that the obstruction is still early in its course. The most common causes of SBO are postopera ve adhesions (60%). The most common procedures that result in adhesions are appendectomy, colorectal surgery, gynecologic procedures, and upper gastrointestinal procedures. Other e ologies of SBO include malignant tumors (20%), hernias (10%), inammatory bowel disease (5%), and volvulus (3%). From an emergency medicine standpoint, the most important steps are early diagnosis and treatment. Left untreated, SBOs result in high mortality and morbidity rates. Patients with early SBOs often present with nausea and diarrhea. As the obstruction progresses, the patient will likely start vomiting, become febrile, and stop passing stool or flatus. Early SBOs can be missed at abdominal radiography; therefore, if the suspicion is high, surgical consultation and CT scanning are mandatory.

sp ot. c

om

ABDOMINAL PAIN, VOMITING, AND DIARRHEA

3o clinical cases from eMedicine Case 17

Background

What is the diagnosis? Hint

Note the episodic abdominal pain and positive radiographic findings.

ww w. me

The patient has a normal birth history, no medical problems, and no prior surgeries. Her immunizations are current.On physical examina on, the girl's vital signs are as follows: rectal temperature, 100.2F; pulse, 134 beats per minute; respira ons, 32 breaths per minute; and oxygen satura on with pulse oximetry, 98% on room air. The patient is alert and sitting comfortably with her parents. Her skin and mucous membranes are moist. The remaining findings are normal and reveal a soft, nontender, nondistended abdomen without rebound or guarding. Rectal examination is not performed. While you speak to the parents, the patient clutches her abdomen and cries, doubling over in pain. The episode spontaneously resolves after several minutes.

dic

alm cq sd ow

nlo

Figure 1

ad .bl og

The episodic symptoms cause the patient to double over in pain and have been increasing in frequency, now occurring every 20 minutes. The pa ent vomits a er every feeding, including liquid feedings, and her last bowel movement occurred 24 hours prior to her admission. She was evaluated 2 days ago for similar complaints and discharged home with a diagnosis of gastroenteritis.

sp ot. c

A 2-year-old girl presents to the pediatric emergency department because of epigastric abdominal pain for the last 3 days associated with fever; vomi ng; and watery, brown stools. Her parents deny hematemesis or blood in her stool.

om

ABDOMINAL PAIN AND VOMITING IN A YOUNG GIRL

3o clinical cases from eMedicine Case 17 Answer

Diagnosis: Intussusception Discussion:

The abdominal radiograph and barium enema study depict an ileocolic intussusception, which is demonstrated by the acute intraluminal colonic filling defect. Intussusception is a process in which a segment of intestine invaginates, or telescopes, into the adjoining intestinal lumen, causing bowel obstruction. It is the most common cause of bowel obstruction in children younger than 6 years. The classic triad of vomi ng, abdominal pain, and passage of curra t jelly stools occurs in n less than one third of pa ents. Currant jelly stool occurs in 60% of cases and is caused by the obstruc on of venous return (Wyllie, 2004). If the intussusception is not reduced, the passage of bloody, mucoid stools; weakness; fever; and shock follows. An upper respiratory infection often precedes these symptoms. The typical patient is usually a previously well child presenting with recurrent severe paroxysmal colicky abdominal pain. Correlations have been noted in patients with adenovirus infections and in infants receiving the tetravalent rhesus-human rotavirus vaccine within 2 weeks of intussuscep on (odds ra o, 21:7) (Wyllie, 2004). Inamed Peyer patches in the terminal ileum secondary to GI infection or the introduction of new food proteins are other postulated mechanisms that might be responsible for creating the lead point in the telescoping process. Other causes for lead points are polyps, Meckel diverticulum, neurofibroma, intestinal duplication, hemangioma, malignancy, and cys c brosis. Lead points are most common in pa ents older than 2 years. Postopera ve intussuscep on usually occurs within 5 days of an abdominal opera on. In rare circumstances, the parents report 1 or more previous a acks of abdominal pain 10 days to 6 months prior to the current episode. Strangula on of the bowel does not usually occur within the rst 24 hours of presenta on but may occur later, leading to intestinal gangrene and shock. The differential diagnosis includes malrotation of the bowel, chronic constipation, Hirschsprung disease, appendicitis, inflammatory bowel disease, Henoch-Schnlein purpura, intestinal vascular malformations, peptic ulcer with bleeding, and portal hypertension with bleeding varices. Management consists of fluid resuscitation, placement of a nasogastric tube to minimize the risk of vomiting and aspiration, preoperative laboratory studies, abdominal radiography, and surgical consultation. In the absence of perforation, administering a barium or air enema can confirm the diagnosis and hopefully reduce the intussusception. A surgeon must be ready to operate if this procedure is not successful. The patient should be observed in the hospital for 24 hours or at home if the parents are reliable and if the pa en can quickly t return if symptoms recur. The recurrence rate for an intussuscep on is 10% a er reduc on with barium enema and 2-5% a er surgical resec on (Wyllie, 2004). Radiographic findings are often normal in intussusception. Early radiographic evidence of an absence of air in the right lower quadrant is known as the Dance sign (Hostetler, 2002). So -tissue opacity in the right upper quadrant may be present in 25-60% of pa ents. Radiographic ndings illustra ng small -bowel dilatation and airfluid levels may represent a small-bowel obstruction. If generalized distention with air-fluid levels is present in the colon, acute gastroenteritis is the more likely diagnosis.

ww w. me

dic

alm cq sd ow

nlo

ad .bl og

sp ot. c

om

ABDOMINAL PAIN AND VOMITING IN A YOUNG GIRL

3o clinical cases from eMedicine Case 18

Background

Hint What is the PR interval?

ww w. me

dic

alm cq sd ow

Figure 1

nlo

ad .bl og

sp ot. c

A 75-year-old man presents to the hospital with a cut on his forehead. He states that he must have fallen but does not know why. He has a history of hypertension and diabetes mellitus.

om

SYNCOPE

3o clinical cases from eMedicine Case 18 Answer

Diagnosis: Third-degree atrioventricular (AV) block Discussion:

ww w. me

dic

alm cq sd ow

nlo

ad .bl og

This is a condition in which all supraventricular impulses are blocked at the AV junction and therefore prevented from reaching and activating the ventricles. As a result, another intrinsic pacemaker activates the ventricles. The supraventricular rhythm, which is usually faster, is completely disassociated from the slower junc onal escape rhythm of less than 40 beats per minute or a slow idioventricular rhythm of less than 35 beats per minute. Complete AV dissociation does not always result in complete AV block. Generally, in AV dissociation, complete heart block is present if the atrial rate is greater than the ventricular rate, whereas heart block is not present if the ventricular rate is greater than the atrial rate (eg, accelerated junctional rhythm, accelerated idioventricular rhythm [AIVR], ventricular tachycardia [VT]).

sp ot. c

om

SYNCOPE

3o clinical cases from eMedicine Case 19

Background A 48-year-old patient presents to the hospital with syncope and hypotension.

Hint

Note the S wave in lead I and the Q wave in lead III.

ww w. me

dic

alm cq sd ow

Figure 1

nlo

ad .bl og

sp ot. c

om

SYNCOPE AND HYPOTENSION

3o clinical cases from eMedicine Case 19 Answer

Diagnosis: Acute pulmonary embolism Discussion:

Sinus tachycardia, the most common ECG abnormality, and the classic S1Q3T3 pa ern are observed. Evidence of right heart strain, a right bundle branch block (RBBB) pattern, and axis deviation is also present.

ww w. me

dic

alm cq sd ow

nlo

ad .bl og

sp ot. c

om

SYNCOPE AND HYPOTENSION

3o clinical cases from eMedicine Case 20

Background

What is the diagnosis? Hint

ww w. me

dic

Children with this idiopathic syndrome often complain of abdominal pain. In some, intussusception or renal dysfunction may even develop. When pressure is placed on the skin lesions, they are palpable but do not blanch.

alm cq sd ow

Figure 1

nlo

ad .bl og

sp ot. c

One week ago, the mother of this 4-year-old girl noted small red dots on the child's legs, which the primary pediatrician diagnosed as bug bites. The lesions became larger, and a schoolteacher confronted the mother about what she thought were bruises on the child's legs. Concerned, the mother brought the child to a dermatologist, and biopsy was performed; however, the mother states that no diagnosis was made. Now, nearly hysterical, the mother brings her daughter to the emergency department and complains that the rash looks worse and that the child refuses to walk. The girl is afebrile, appears well, and she is able to ambulate.

om

RASH ON A CHILD'S LEGS

3o clinical cases from eMedicine Case 20 Answer

Diagnosis: Henoch-Schonlein purpura (HSP), anaphylactoid purpura Discussion:

ww w. me

dic

alm cq sd ow

nlo

ad .bl og

The classic syndrome of HSP consists of a purpuric rash, arthritis, gastrointestinal symptoms, and renal involvement. Skin lesions are required for the diagnosis and often appear as small wheals or erythematous macules that progress to purpura. Angioedema of the face, hands, feet, and perineum is also common. Two thirds of the patients have arthritis in the large joints, and more than half have gastrointestinal symptoms. Colicky abdominal pain and vomiting are typical. Hemoccult results may be positive, or the stool can be grossly bloody. The failure to recognize HSP can result in unnecessary laparotomy; however, intussusception or smallbowel obstruc on rarely complicates HSP. Approximately 2550% of the pa ents have renal involvement. Hematuria and proteinuria are the most common findings, but patients can have hypertension, azotemia, oliguria, or nephrotic syndrome as well. Central nervous system involvement is extremely rare. Most patients have a benign course and require only nonsteroidal anti-inflammatory drugs (NSAIDs) for symptomatic relief. Relapses may occur. Patients with intussusception, small-bowel obstruction, or nephrotic syndrome are given prednisone 1-2 mg/kg/d and require hospitaliza on.

sp ot. c

om

RASH ON A CHILD 'S LEGS

3o clinical cases from eMedicine Case 21

Background A 32-year-old man presents to the hospital with chest pain that began at a party.

Figure 1

Hint He is extremely young for atherosclerotic disease.

ww w. me

dic

alm cq sd ow

nlo

ad .bl og

sp ot. c

om

CHEST PAIN AT A PARTY

3o clinical cases from eMedicine Case 21 Answer

Diagnosis: Acute anterior-septal myocardial infarction Discussion:

Note the ST-segment eleva on in leads V1 through V4. Before his chest pain started, the pa ent was using cocaine, which induced coronary vasospasm. This was his fifth such emergency department visit, all of which had a similar clinical presentation.

ww w. me

dic

alm cq sd ow

nlo

ad .bl og

sp ot. c

om

CHEST PAIN AT A PARTY

3o clinical cases from eMedicine Case 22

Background

Which of the following tests would be most helpful in the diagnosis of this case? A. B. C. D. E. Hint The diagnosis is not idiopathic depression. Liver function tests Complete blood count analysis Thyroid-stimulating hormone (TSH) test Urine toxicology screening Rapid plasma reagin (RPR) test

ww w. me

dic

alm cq sd ow

nlo

ad .bl og

sp ot. c

A 65-year-old white woman presents with depression, excessive sleepiness, chronic malaise, emotional lability, constipation, muscular weakness, imbalanced gait, and arthralgias. Two years ago, after the death of her husband of 45 years, she immigrated to the United States from Copenhagen, Denmark, to live with her son and his family. Her medical, surgical, and family histories are normal except for approximately 1 tension headache per month, which she successfully treats with 2 over-the-counter ibuprofen tablets. Her family physician has treated her with uoxe ne (Prozac) for 8 months, without improvement in her symptoms.

om

DEPRESSION IN A 65-YEAR-OLD WOMAN

3o clinical cases from eMedicine Case 22 Answer

Diagnosis: Hypothyroidism Discussion:

ww w. me

dic

alm cq sd ow

nlo

ad .bl og

The correct answer is C, a thyroid-stimulating hormone (TSH) test: This patient has depression and other findings secondary to hypothyroidism. All of the tests listed are part of the diagnostic workup in patients in whom depression is suspected. However, in this patient, the TSH measurement would be most helpful because depression is common in hypothyroidism. This patient has spent most of her life in Denmark. Hypothyroidism is much more common in Europe than in the United States, as a result of the prevalence of iodine deficiency there. Several studies have shown that the prevalence of hypothyroidism in Europe may be as high as 2-5%. In addition to depression, other neurologic sequelae can result from hypothyroidism. These include myopathy, which caused this patient^s weakness, and neuropathy, which contributed to her imbalanced gait.

sp ot. c

om

DEPRESSION IN A 65-Y EAR -O LD WOMAN

3o clinical cases from eMedicine Case 23

ww w. me

dic

On physical examina on, his vital signs demonstrate a strong pulse, with a regular rhythm and a rate of 58 bpm, blood pressure of 123/60 mm Hg, weight of 165.3 lb (75 kg), and oral temperature of 98.1F (36.7C). He is a very fit, well-developed white male in no apparent distress. The pharynx shows no erythema or exudate, and the neck examination demonstrates no tenderness to palpation or lymphadenopathy. The cardiac examination is notable for a point of maximum intensity at the 5th le interspace, but nomurmurs, gallops, or rubs are appreciated. The patient's pulses are strong bilaterally. The respiratory examination reveals lungs clear to auscultation bilaterally. The abdominal examination is unremarkable, and the stool test is negative for occult blood. The skin examina on show lesions ranging in diameter from 2 mm to 10 cm (see Figures 13). The macules and plaques are nonblanchable, and most of them are concentrated in the posterior calves, the palms,

alm cq sd ow

Figure 1

nlo

A 20-year-old active-duty male soldier presents to the emergency department (ED) of a military hospital complaining of a 3-day history of a dark-red "burning rash." The rash started at his sock line and, over the course of the past 2 days, has spread proximally up his thighs. It is not present on his abdomen or back, but it has spread to his hands over the past day. The patient also developed a sore throat and a scratchy voice the day before presentation, without odynophagia. He was in the North Carolina woods as part of his infantry training, which is supplemental training after boot camp. He reports having spent 20+ hours per day for the past several days in a foxhole (a hole in the ground that soldiers use for protection). The patient reports fatigue only related to his level of activity and lack of sleep during training. He is not taking any medications, he has no allergies, and he has smoked 5-10 cigare es daily for the past year. He also reports drinking approximately 510 beers with his comrades 1-2 mes per month before star ng his training. The pa ent notes some right ankle swelling and pain that started a er a 12.43 mi (20 km) eld hike with a 60 lb (27.22 kg) rucksack on his back. The pain and swelling began 1 month before admission and has worsened in the past few days. He has had no pruritus, fevers, chills, abdominal pain, diarrhea, changes in urine color, or dysuria, as well as no new sexual contacts and no recent animal or insect bites. No one else in his military unit has reported similar skin findings. The patient had a minor motorcycle accident 18 months ago, with a right leg lacera on that required suture repair. He has no significant family history.

ad .bl og

sp ot. c

Background

om

A 20-YEAR-OLD SOLDIER WITH A 3-DAY HISTORY OF A DARK, BURNING RASH

3o clinical cases from eMedicine


and the soles of the feet. No macules or plaques are noted on the face, chest, or back. The lesions are not tender to palpa on or warm to the touch. His hands and right ankle exhibit 1+ nonpi ng edema. The pa ent has a scar on his right leg from his motorcycle accident that is well-healed, with no fluctuance or erythema. The neurologic examination is unremarkable.

A urinalysis shows 2+ protein and 2+ blood, with 25-50 red blood cells (RBCs) per high-power field on microscopy. His coagulation studies at admission include a prothrombin me (PT) of 12.3 s, an interna onal normalized ra on (INR) of 1.08, and a par al thromboplas n me (PTT) of 25.5 s. The pa ent's blood urea nitrogen (BUN) is 24 mg/dL (8.59 mmol/L) and his crea nine value is 1.1 mg/dL (97.2 mol/L). The complete blood count (CBC) shows a white blood cell (WBC) count of 7.4 103/L (7.4 109/L), a hemoglobin (HGB) of 12.4 g/dL (124 g/L), a hematocrit (HCT) of 37.1% (0.371), and a platelet count of 195 103/L (195 109/L), with a normal smear. Skin punch biopsies of 2 of the lesions are obtained that demonstrate small-vessel leukocytoclastic vasculitis. Immunofluorescence of the skin biopsies demonstrates a weak linear pattern at the dermal-epidermal junction for immunoglobulin G (IgG), immunoglobulin A (IgA), and complement 3 (C3).

ww w. me

dic

alm cq sd ow

Figure 3

nlo

Figure 2

ad .bl og

sp ot. c

om

3o clinical cases from eMedicine


What is the diagnosis? Hint: The patient is presenting with nonblanchable lesions after reporting an antecedent sore throat. o o o o o Henoch-Schnlein purpura (HSP) Mixed cryoglobulinemia Polyarteritis nodosum Systemic lupus erythematosus Hemolytic uremic syndrome (HUS)

ww w. me

dic

alm cq sd ow

nlo

ad .bl og

sp ot. c

om

3o clinical cases from eMedicine o Case 23 Answer

Diagnosis: Henoch-Schnlein purpura (HSP) Schnlein Discussion:

The sine qua non for the diagnosis of Henoch Schnlein purpura is a skin biopsy finding of IgA deposition at the Henoch-Schnlein dermal-epidermal junction. A skin biopsy with immunofluorescence studies is recommended if clinical suspicion epidermal of Henoch-Schnlein purpura exists. The etiology of the disease remains unknown, but it is understood to be an Schnlein autoimmune response (usually to an upper respiratory infection). Other items in the differential diagnosis include many of the etiologies of palpable purpura. An algorithmic a empt has been made (see Table 1) to delineate these e ologies, but classica on remains dicult.Palpable purpura is a classic skin manifestation of cutaneous vasculitis, but there are many other entit that can cause entities it. Additional information can be gleaned from the pathologic diagnosis of small vessel vasculitis, which also carries its own dieren al (see Table 2). The intersec on of these 2 dieren als consists of the following set of diagnoses: mixed cryoglobulinemia, vasculitis associated with collagen vascular disease (ie, systemic lupus es: erythematosus, rheumatoid arthritis), and Henoch Henoch-Schnlein purpura. Mixed cryoglobulinemia was easily ruled out, as the patient's serum cryoglobulin test a complement levels and were a C3 of 118 mg/dL (1.18 g/L; normal range, 79 152 mg/dL), a C4 of 26 mg/dL (0.26 g/L; normal range, 16 79-152 1638 mg/dL), complement CH50 of 53.0 U/mL (53 kU/L; normal range 22 60 U/mL), and a nega ve serum 22-60 cryoglobulin examination. Another diagnosis that was readily excluded was systemic lupus erythematosus. Our her pa ent had only 1 arthralgia, but no fevers, myalgia, or malaise were noted; only 1/11 American Rheumatologic Association criteria for the diagnosis of lupus were met. To satisfy the clinical diagnosis of systemic lupus satisfy erythematosus, 4/11 American Rheumatologic Associa on criteria are required. Although the pa ent meets the histopathologic criteria for solitary IgA nephropathy, as the renal biopsy showed diffuse proliferative endocapillary glomerulonephritis with increased mesangial matrix and cellularity, capillary wall thickening, and lobular accentua on on light and electron microscopy with strong (3+) mesangial staining for IgA and fibrinogen, as well as lappa and lambda l light chains (2-3+) present on immunouorescence microscopy, he also 3+) exhibited some of the cardinal clinical features of Henoch Schnlein purpura at presentation. He denied having Henoch-Schnlein

ww w. me

dic

alm cq sd ow

There exist several guidelines for the diagnosis of Henoch Schnlein purpura. The American College of Henoch-Schnlein Rheumatology (ACR) requires 4 criteria (published in 1990) for diagnosing Henoch-Schnlein purpura: palpable es Henochpurpura, pa ent age of 20 years or less at onset, bowel angina, and the presence of granulocytes in the vessel walls. More recently, in 2006, the European League Against Rheumatism (EULAR) and the Pediatric Rheumatism Rheumatology Society published their own Henoch Schnlein purpura criteria. These include palpable purpura Henoch-Schnlein as a mandatory criterion, with at least one of the following conditions: diffuse abdominal pain, predominant IgA deposition (confirmed on biopsy), acute arthritis in any joint, and renal involvement (as evidenced by hematuria and/or proteinuria). According to the older ACR criteria, the diagnosis of Henoch Henoch-Schnlein purpura for our patient could be called into question, as he exhibited no bowel angina; however, according to the more recent on, EULAR criteria, our patient fits the diagnosis of Henoch Schnlein purpura quite well, as he has palpable Henoch-Schnlein purpura, predominant IgA deposition confirmed on biopsy, a monoarticular arthralgia in his right ankle, and arthralgia renal involvement.

nlo

ad .bl og

sp ot. c

om

A 20-YEAR-OLD SOLDIER WITH A 3-DAY HISTORY OF A DARK, BURNING RASH

3o clinical cases from eMedicine


had blood in his urine on prior urinalyses or a history of gross blood urine, lessening the possibility of IgA nephropathy. After further inquiry, it was confirmed that the patient had no family history of renal disease and, as such, it is unlikely that he has familial IgA nephropathy. Polyarteritis nodosum (PAN) could be considered in the differential diagnosis, but this patient displayed none of the classic signs or symptoms of fever, abdominal pain, nausea, and weight loss. Also common to PAN is the is involvement of small- and medium-sized arteries, as well as subcutaneous nodules (not palpable purpura). Joint pain, however, is very common in PAN. PAN may be associated with hepatitis B or C, but this patient's hepatitis panel was negative. A hemorrhagic diathesis was quickly ruled out with the normal coagulation studies. Lyme disease, in contrast to Henoch-Schnlein purpura, more commonly presents with the typical target lesions of erythema migrans and a positive Lyme titer (and not with a purpuric rash). This patient had a negative Lyme antibody examination, although this finding alone does not exclude the possibility of the disease. Multiple arthralgias are more common in Lyme disease, and they seldom present with abdominal pain. Hemolytic uremic syndrome (HUS) should be considered in the differential diagnosis, but it can be easily recognized by the presence of microangiopathic anemia and is distinguished from Henoch-Schnlein purpura by the absence of purpura and arthralgias. Henoch-Schnlein purpura is an autoimmune, self-limited IgA-mediated vasculitis that most commonly presents in children 5 years of age. In fact, 95% of cases present in children <10 years of age. There is a 2:1 distribu on between males and females. The overall incidence is 0.14 cases per 1000 popula on in children aged 214. This patient is in a relatively uncommon demographic for the disease. The most common signs and symptoms of the disease are palpable purpura (100%), abdominal pain (63%), gastrointes nal bleeding (33%), arthralgia (82%), nephri s (40%), and hematuria. Risk factors associated with apoor prognosis for renal survival include nephrotic syndrome, decreased factor XIII activity, hypertension, and renal failure at onset. It is known that severe Henoch-Schnlein purpuraassociated nephritis and progression to nephrotic syndrome are associated with a poorer prognosis.

Our patient was admitted to the hospital ward and vigorously volume-expanded with normal saline at 250 cc/h. His crea nine clearance was 182.7 mL/min (3.05 mL/s; this is within normal limits). Also notable was his an streptolysin O (ASO) ter of 722 Todd U/mL (normal range,-117 Todd U/mL). Even with his severe skin 0 manifestations, the severity of our patient's Henoch-Schnlein purpura nephritis on admission was low. The patient's positive ASO titer pointed to a recent presumed streptococcal pharyngitis. The skin biopsy report was significant for small-vessel leukocytoclastic vasculitis. The immunofluorescence study of the sample demonstrated a weak linear pattern at the dermal-epidermal junc on, with IgG, IgA, and C3, which was consistent with the diagnosis of Henoch-Schnlein purpura. On admission, the patient's protein-to-creatinine ra o was 5.5, consistent with nephrotic-range proteinuria, or >3.5 grams of protein per day. A normal proteinto-crea nine ra o is <0.2. Although the nephrology service was consulted early in the clinical course and the patient was started on high dose intravenous methylprednisolone therapy at 1 gram divided twice a day on hospital day 1, his condi on rapidly worsened. By hospital day 4, the pa ent's serum crea nine was 1.1 mg/dL (97.2 mol/L) and his protein-to-crea nine ra o had increased to 15.1. His urine volume was 1850 on that mL day, with 24 h protein and crea nine excre on totaling 35.8 grams and 2384 mg, respec vely. The pa ent, while still normotensive, had developed prominent edema consistent with nephrotic syndrome. As his renal status had worsened significantly, he was transferred to a tertiary care center for further nephrology care. A subsequent renal biopsy demonstrated diffuse proliferative endocapillary glomerulonephritis consistent with IgA nephropathy, or Henoch-Schnlein purpura. Mild chronic tubulointerstitial disease and mild hyaline

ww w. me

dic

alm cq sd ow

The mainstays of treatment for Henoch-Schnlein purpura are observation, intravenous hydration, and steroid therapy. Most cases are self-limited and resolve within 4 weeks. Cyclophosphamide is usually reserved for severe, systemic forms of vasculitis, and appropriate consultation with a nephrologist is recommended. Chronic kidney disease and end-stage renal disease are known complications of Henoch-Schnlein purpura, with incidence rates of 5% and 1.5%, respec vely. It shold be noted that corticosteroids should only be instituted u a er consulta on with a nephrologist, as they are of ques onable benet. In fact, only 1 randomized controlled clinical trial has been performed evaluating the use of corticosteroids in treating Henoch-Schnlein purpura as renal protective agents. Huber et al found that early prednisone therapy initiation did not ward off chronic kidney disease at 1 year; however, there has been a case report of successful treatment of an adult pa ent with crescentic glomerulonephritis (from Henoch-Schnlein purpura) with 9 rounds of double-filtration plasmapheresis and concomitant plasma replacement.

nlo

ad .bl og

sp ot. c

om

3o clinical cases from eMedicine


arteriolar sclerosis were noted as well. A regimen of cyclophosphamide was initiated, and the patient deteriorated to end-stage renal disease by the time this case was compiled.

Intravascular (usually nonpalpable)

Vascular (usually palpable)

Inflammatory vascular causes


Extravascular and miscellaneous (may be palpable or nonpalpable)

dic

ww w. me

alm cq sd ow

Cutaneous vasculitis Purely cutaneous vasculitis (eg, secondary to medications) Henoch-Schnlein purpura Polyarteritis nodosa Granulomatous vasculitis (Wegener granulomatosis, Churg-Strauss vasculitis) Cutaneous vasculitis associated with a collagen Vascular disease (eg, systemic lupus erythematosus, rheumatoid arthritis) Giant cell arteritis Mixed cryoglobulinemia Hyperglobulinemic purpura Trauma Subacute bacterial endocarditis Other embolic diseases Amyloidosis Corticosteroids Toxins and venoms Senile purpura Scurvy Valsalva maneuver Pseudopurpura (Sweet syndrome, cherry angiomas, angiokeratoma, Kaposi sarcoma)

Noninflammatory vascular causes


nlo

ad .bl og

Alterations in platelet formation, destruction, or function Drugs (aspirin, methyldopa) Idiopathic thrombocytopenic purpura Thrombotic thrombocytopenic purpura Disseminated intravascular coagulation Infection (especially Neisseria, Rickettsia, Staphylococcus) Splenic sequestration Radiation therapy Myelofibrosis Myeloproliferative disorders

sp ot. c

Table 1: E

ology of Purpura

om

3o clinical cases from eMedicine


Table 2: Clinical Signs of Necro Small Vessels Signs Urticaria Palpable Purpura Diseases

zing Vasculi

s with Respect to Vessel Size Involved

Henoch-Schnlein purpura Essential Mixed Cryoglobulinemia Vasculitis associated with connective tissue disease Vasculitis associated with malignancy Serum sickness and serum sickness-like reactions Chronic urticaria (urticarial vasculitis) Urticarial prodrome of acute hepatitis type B infection

Nodules, bullae, or ulcers Large Vessels Signs Subcutaneous ecchymoses nodules, ulceration, and,

Diseases

ww w. me

dic

alm cq sd ow

nlo

Polyarteritis nodosa Churg-Strauss syndrome Wegener granulomatosis Giant cell (temporal) arteritis

ad .bl og

sp ot. c

Hypersensitivity vasculitis

om

3o clinical cases from eMedicine Case 24

Background

dic

Today, while preparing to board a plane, he developed a worsening headache, bilious emesis, palpitations, and sweats. He decided to delay his trip and is now presenting to the local ED for further evaluation. The patient denies having any trauma, seizures, abdominal pain, stiff neck, or photophobia. He has no significant past medical history, and his only medica on use is the ibuprofen that he has taken over the past 2 days. On physical examina on, his temperature is 103.0 F (39.4 C), his pulse is 83 bpm, his blood pressure is 120/70 mm Hg, his respiratory rate is 16 breaths/min, and his oxygen satura on is 96% while breathing room air. The patient is generally well-appearing, alert, and oriented. The examination of the head, eyes, and pupils is unremarkable. The neck is supple, without any lymphadenopathy. On auscultation, the lungs are clear; additionally, the patient's heart has a regular rate and rhythm, without any murmurs. The examination of the abdomen reveals normal bowel sounds, with mild tenderness to palpation in the right and left upper quadrants. The spleen is noted to be 3 cm below the costal margin. The neurologic examina on is unremarkable.

ww w. me

alm cq sd ow

nlo

Figure 1

ad .bl og

sp ot. c

A 17-year-old man presents to the emergency department (ED) complaining of a headache that has lasted for the past 2 days. The pa ent states that he returned to the United States 2 days ago a er spending 3 weeks in Nigeria. According to the patient, he felt well when he initially arrived in the United States, but he developed a severe headache soon after. The headache is constant and throbbing, is lasting throughout the day, and is relieved with ibuprofen. He complains of subjective fevers and intermittent sweats, especially at night, but he has not taken his temperature.

om

A TRAVELERS FEVER

3o clinical cases from eMedicine


The patient is treated with intravenous fluids. An electrocardiogram (ECG) shows a normal sinus rhythm, with a QTc of 390 msec. Serum laboratory tests are done, and they are signicant for the following values: crea nine, 1.2 mg/dL (106.08 mol/L); magnesium, 1.2 mg/dL (0.49 mmol/L); alanine aminotransferase (ALT), 110 U/L (normal range 0-35 U/L); aspartate aminotransferase (AST), 159 U/L (normal range 0-35 U/L); total bilirubin, 2.7 mg/dL (46.17 mol/L) (normal range <1.20 mg/dL); direct bilirubin, 0.9 mg/dL (15.39 mol/L) (normal range <0.20 mg/dL); and alkaline phosphatase, 152 U/L (normal range 30-130 U/L). A complete blood count (CBC) reveals a white blood cell (WBC) count of 5.7 103/L (5.7 109/L), a hematocrit of 44 % (0.44), and platelets of 34 103/L (34 109/L). A blood smear is obtained (see Figure 1). What is the diagnosis? Hint: Pay close attention to the finding of the blood smear. o o o o Malaria Babesiosis Lyme Disease Ehrlichiosis

ww w. me

dic

alm cq sd ow

nlo

ad .bl og

sp ot. c

om

3o clinical cases from eMedicine o Case 24 Answer

Discussion:

Malaria results from an infec on caused by any of the following 4 protozoa of the genusPlasmodium: falciparum, vivax, ovale and malariae. Transmission of the parasite occurs via the bite of the Anopheles malariae. mosquito. Once the protozoa are injected into the bloodstream, they enter the hepatic cells and reproduce; re eventually, the hepatic cells erupt and release more protozoa into the host's circulation. These parasites then remain in the bloodstream, periodically invading erythrocytes, causing hemolysis, and infecting new RBCs. causing The incuba on period tends to be 918 days for P falciparum, P vivax and P ovale but P malariae has an ovale, incuba on period of 1840 days. The most common parasites seen in the US are P falciparum, which is often found in travelers returning from Sub d Sub-Saharan Africa, and P vivax, which is found in those returning from Asia, , Eastern Europe, and La n America. The clinical presenta on also varies between these 2 parasites:falciparum P often causes symptoms within the first month following the travel period, and it can be fatal; of patients first infected with P vivax, 50% have symptoms within 1 month a er travel, and approximately 2% of pa ents may , have symptoms 1 year a er exposure. The majority of pa ents infected with ither parasite are usually e symptoma c within the rst 3 months a er they return to the US. The clinical presentation of malaria can vary widely and depends on the species of Plasmodium involved. Common symptoms include fever, malaise, myalgias, and headache, which may be accompanied by cough, headache, abdominal pain, or diarrhea. Since these symptoms are non specific, malaria should be considered in all febrile non-specific, travelers, regardless of their clinical presenta on. In fact, approximately 78100% of pa ents presening with t malaria are febrile when they are first examined. The classically described fever patterns are rarely observed; however, when these fevers do occur at 48 to 72-hour intervals, this finding is virtually pathognomonic for P 48hour vivax, P ovale, and P malariae infections. This cyclical pattern of symptoms coincides with the regular interval of erythrocyte hemolysis. On examina on, splenomegaly is found in 2448% of pa ents, and pa ents may complain of abdominal pain. Severe malaria, usually caused by P falciparum, causes several manifestations, , including prostration, impaired consciousness or coma, respiratory distress caused by pulmonary edema and acute respiratory distress syndrome (ARDS), seizures, circulatory collapse, abnormal bleeding (including

ww w. me

dic

alm cq sd ow

Worldwide, there are about 300-500 million new cases of malaria annually. Malaria is the most deadly vector 500 vectorborne illness in the world, causing 3.5 5 million deaths annually. On average, 40% of the 50 million people who 3.5-5 travel from industrialized to developing countries each year report some illness associated with their travel. developing Approximately 1,200 cases of malaria are reported each year in the United States among travelers; therefore, it is important to consider malaria as a possible cause of fever in the returning traveler.

nlo

This case is an example of malaria caused by Plasmodium falciparum. In the context of the patient's recent . travel to Nigeria and presentation with fever, malaria was strongly suspected. The patient's blood smear was notable for 15% parasitemia, most likely P falciparum (as evidenced by the circles within some of the red blood cells [RBCs] in the smear seen in Figure 2). The other laboratory ndings, such as the thrombocytopenia and elevated bilirubin, correlated with the disease burden. The elevated creatinine value was also noted, as this creatinine finding is sometimes seen in more aggressive cases of the disease; however, this usually resolves with time.

ad .bl og

Diagnosis: Malaria

sp ot. c

om

A TRAVELERS FEVER

3o clinical cases from eMedicine


disseminated intravascular coagulopathy), splenic rupture, jaundice, severe anemia, acute renal failure, and acidosis. The level of parasitemia o en exceeds 5%. The diagnosis of malaria is made by examination of both thin and thick blood smears. These smears are used to quantify the level of parasitemia, which is used to guide treatment. If the first smear is negative, it should be repeated at 12- to 24-hour intervals for 48-72 hours. If the diagnosis is clinically suspected and a sucient laboratory diagnosis is not possible, empirical treatment for P falciparum should be started because the disease can be fatal if left untreated. Other laboratory findings, such as normocytic anemia, thrombocytopenia, low WBC count, elevated lactate dehydrogenase, and elevated bilirubin, are nonspecific, but they may provide clues to the diagnosis. The treatment options for malaria vary according to the region where the traveler was likely to have acquired the infection. This is based on the local prevalence and antimalarial drug susceptibility of particular Plasmodium species. Chloroquine is the treatment of choice for those patients infected with P vivax, P ovale, and P malariae. Patients infected with either P vivax or P ovale must also take primaquine, as these species may lie dormant within the liver and are not eradicated by chloroquine alone. Failure to treat P vivax and P ovale with both agents frequently leads to recrudescence of the disease. If P falciparum has been eliminated as the causative agent, patients may be treated in the ambulatory setting. If P falciparum is suspected, treatment should be ini ated as soon as possible, as severe illness or death can occur in as li le as 1 days a er presenta n. The -2 o CDC recommends hospitalization in order to ensure that patients are able to tolerate oral therapy and have an appropriate response to treatment. For those patients returning from areas where Plasmodium species have known chloroquine resistance, such as Southern Asia, Sub-Saharan Africa, and Oceania, there are 3 available treatment op ons: (1) oral quinine along with tetracycline, doxycycline, or clindamycin; (2) atovaquone proguanil; or (3) meoquine along with doxycycline. Depending on the severity of the disease and the region where the parasite was acquired, dierent lengths of treatment are recommended; these vary from 3 to 7 days of quinine therapy and 7 days of an bio cs. For those pa ents with severe malaria, an ini al course of doxycycline with quinidine, an intravenous isomer of quinine, is recommended un l the parasite burden is <1%, at which point oral therapy should be initiated. Unfortunately, quinine and quinidine have several side effects, and patients must be closely monitored for cardiac dysrhythmias caused by prolongation of the QTc, profound asymptomatic hypoglycemia, and respiratory distress. In this case, the diagnosis of malaria resulting from P falciparum infection was promptly made based on the patient's travel history and the severity of his symptoms. He was admitted to the pediatric intensive care unit for close observation because of the potentially dangerous side effects of treatment. The patient received a bolus of intravenous quinidine and clindamycin. His parasitemia was <0.5% on the second day of treatment, and the patient was switched to oral quinine therapy and clindamycin. His platelet and creatinine levels improved on hospital day 5, and he was discharged to home on oral therapy for a total of 7 days.

ww w. me

dic

alm cq sd ow

nlo

ad .bl og

sp ot. c

om

3o clinical cases from eMedicine Case 25

Background

The pain started about 4 hours before presenta on to the ED and has been persistent; it is present in the upper abdomen and is centered in the epigastrium. He describes the pain as deep and burning. There is no associated nausea or vomiting. He does not report any changes in his bowel habits and has not experienced any recent fevers. The review of systems is also negative for any recent unintended weight loss or trauma. The patient also reports having had "indigestion" in the past that caused pain similar to what he is currently experiencing, though much less in intensity. His past medical history is significant for coronary artery disease and hypertension. He takes two medications, both for his high blood pressure, but does not drink excessively and does not smoke. On physical examination, the patient is pale and in obvious severe discomfort. His heart rate is 122 bpm, and his blood pressure is 110/65 mm Hg. He is breathing with rapid shallow breaths at a rate greater than 30 breaths/min. His temperature is normal at 99.2F (37.3C), and a pulse oximetry reading while the patient is breathing room air shows a satura on rate of 100%. The cardiovascular and respiratory ndings are unremarkable. The patient has significant tenderness in the epigastric region, with a rigid abdomen. There is little to no tenderness to palpation in the lower quadrants; a reliable assessment of the upper quadrants is not possible because of the tenderness in the epigastric region. Hyperactive bowel sounds are heard on auscultation. The patient's stool is brown and guaiac positive. An electrocardiogram is performed and is noted to be unremarkable except for sinus tachycardia. A complete blood count (CBC) and a chemistry panel are sent. The CBC reveals a mild anemia, with a hemoglobin concentra on of 127 g/L (12.7 g/dL). On the chemistry pane there is evidence of a slight azotemia, with a l, blood urea nitrogen level (BUN) of 17.1 mmol/L (48 mg/dL) and a crea nine value of 106 mol/L (1.2 mg/dL). The remainder of the laboratory investigations are unremarkable. Plain radiographs of the abdomen are performed (see Figures 1A and 1B).

ww w. me

dic

alm cq sd ow

Figure 1

nlo

ad .bl og

sp ot. c

A 76-year-old man presents to the emergency department (ED) complaining of a sudden onset of abdominal pain.

om

ACUTE ONSET OF ABDOMINAL PAIN IN A 76-YEAR-OLD MAN

3o clinical cases from eMedicine


What is the diagnosis? Hint: Both the inner and the outer walls of the bowel are visible. o o o o Abdominal aortic aneurysm rupture Pancreatitis Pneumoperitoneum from duodenal ulcer perforation Acute coronary syndrome

ww w. me

dic

alm cq sd ow

nlo

ad .bl og

sp ot. c

om

3o clinical cases from eMedicine o Case 25 Answer

Discussion:

The underlying pathophysiology of a duodenal ulcer is a common condition that is characterized by the presence of a well-demarcated mucosal defect in the duodenum. Approximately 95% of duodenal ulcers ar demarcated are found in the rst part of the duodenum; most are less than 1 cm in diameter. A prompt and accurate diagnosis combined with treatment can effectively prevent potentially serious complications, such as perforation (which occurred in this case). The duodenal mucosa resists damage from the effects of gastric acid and the proteolytic al enzyme pepsin as a result of the protective qualities of the mucous/gel layer produced by the mucus mucus-secreting epithelial cells, bicarbonate secretions from other gastric and duodenal cellular components, and protective duodenal prostaglandins. If gastric acid and pepsin penetrate the mucous layer and reach the epithelial cells, ion pumps in the basolateral cell membrane regulate intracellular pH by removing excess hydrogen ions; healthy ep epithelial cells migrate to the site of the injury; and mucosal blood flow serves to remove any excess acid diffused through the injured mucosa. Despite these barriers and mechanisms to prevent permanent injury, ulcerations can occur. Any pathophysiologic or iatrogenic process that increases gastric acidity (eg, disease states with increased maximal and basal acid output), decreases prostaglandin production (eg, nonsteroidal anti anti-inflammatory drug [NSAID] use, which leads to inhibition of the cyclooxygenase [COX-1] pathway), or interferes with the mucous cyclooxygenase-1 1] layer (eg, Helicobacter pylori infection leading to stimulation of gastric acid production) can result in the formation of peptic ulcer disease. In the US, the prevalence of duodenal ulcer is es mated to be 615% in the general popula on. The majority of individuals with duodenal ulcers do not have clinically significant disease. The prevalence is closely linked to the presence of H pylori infection. Of those individuals infected with H pylori, the lifetime prevalence is ion. ,

ww w. me

dic

alm cq sd ow

Although many patients with a perforated peptic ulcer initially present with severe abdominal pain as well as epigastric tenderness and classic signs of peritonitis, patients who are elderly, are immunocompromised, or peritonitis, have an altered mental status may have only minimal signs and symptoms. In one study of pa ents >60 years with perforated ulcers, only 70% had abdominal pain. Other pa ents reported symptoms tht included a dyspepsia, anorexia, nausea, and vomi ng. Severe abdominal pain was present in only 16% of pa ents. About 6% of pa ents with perforated ulcers have no abdominal ndings. In most cases of perfora on from ulcer disease, gastric and duodenal content (ie, bile, ingested food, swallowed bacteria) leaks into the peritoneum, resulting in peritonitis and an increased risk of infection and abscess formation. In addition to sepsis, subsequent third-spacing of fluid into the peritoneal cavity caused by the peritonitis can lead to hypotension spacing by and shock.

nlo

Clear deni on of both the inner wall and outer wall of the bowel on Figure 2A (the Rigler sign; see below for explanation) and the presence of free air under the right hemidiaphragm on Figure 2B demonstrate n) pneumoperitoneum. The term "pneumoperitoneum" refers to air in the peritoneal cavity. The differential diagnosis of pneumoperitoneum includes iatrogenic causes (eg, peritoneal dialysis; abdominal surgery, peritoneal including laparoscopy and laparotomy, a er which pneumoperitoneum can persist for up to 28 days), blunt or penetrating trauma, perforation of the hollow viscera (eg, gastric ulcer, duodenal ulcer), pneumatosis intestinalis or pneumatosis coli, vaginal insufflation, and gas from the mediastinum (eg, from barotrauma). lis

ad .bl og

Diagnosis: Pneumoperitoneum from duodenal ulcer perforation

sp ot. c

om

ACUTE ONSET OF ABDOMINAL PAIN IN A 76-YEAR -OLD MAN

3o clinical cases from eMedicine


approximately 20%. Interna onally, the prevalence of the disease varies among countries and, as in the US, is linked to rates of H pylori infection. Duodenal ulcers can result in significant morbidity and mortality. The main complication from these ulcers is pain; however, serious complications can include ulcer hemorrhage, perforation, penetration, and obstruction. Complications and mortality are generally greater in elderly patients than in other patient populations, possibly because of a higher incidence of comorbid disease and an increased use of NSAIDs in this group. When a perforation is suspected, as in this case, an upright chest or left lateral decubitus radiograph can detect as li le as 12 mL of gas under the diaphragm or lateral margin of the liver, especially if strict posi oning techniques are used. Therefore, these studies should be the first diagnostic modalities used. Supine abdominal radiographs are generally of limited value in diagnosing pneumoperitoneum. Computed tomography (CT) scanning is highly sensitive for depicting pneumoperitoneum, and this modality has the added benefit of assisting the clinician in identifying the underlying etiology in many patients. Numerous signs are described for diagnosing pneumoperitoneum on plain radiographs. One of the best known, the Rigler sign (also known as the double-wall or bas-relief sign), is a visualization of the outer surface of a bowel loop wall resulting from free air in the peritoneal cavity. The intraluminal gas provides negative contrast and outlines the internal wall. The cupola sign, typically seen on supine radiographs, is an inverted cup-shaped arcuate lucency overlying the lower thoracic spine and projecting caudally to the heart. This sign is formed as air accumulates anteriorly in the median subphrenic space under the central leaf of the diaphragm. The umbilical ligaments, the urachus, and particularly the falciform ligament are sometimes identified as linear radiopaque structures in the presence of free air. Another common sign is a collection of gas in the right upper quadrant adjacent to the liver, lying mainly in the subhepatic space and in the hepatorenal fossa, that is visible as an oval or triangular gas shadow not in obvious continuity with the rest of the bowel. This collection is usually present in the medial aspect of the right upper quadrant, with a superomedial to inferolateral orientation. The football sign is visualization of the entire peritoneal cavity as an oval gas shadow, with the vertically oriented falciform ligament representing the seam of an American football. This sign is most often seen in the pediatric patient but not in the adult patient, because in the adult patient, there is usually not enough air relative to the size of the peritoneal cavity. Nonsurgical management of a perforated ulcer is associated with prohibitive morbidity and mortality rates, especially in high-risk groups, such as immunocompromised patients and the elderly. Surgical management is generally indicated. Initial management includes gastric decompression with a nasogastric tube, pain control, intravenous hydration, and administration of broad-spectrum antibiotics. Closure with a piece of omentum (Graham patch) and truncal vagotomy with pyloroplasty (by incorpora ng the perfora on) are 2 common approaches to the surgical management of a perforated duodenal ulcer. The patient in this case underwent an omental patch repair for the duodenal perforation.

ww w. me

dic

alm cq sd ow

nlo

ad .bl og

sp ot. c

om

3o clinical cases from eMedicine Case 26

Background

dic

ww w. me

On physical examination, his vital signs are signicant for a respiratory rate of 70 breaths/min and a heart rate of 296 bpm. He is afebrile, with a rectal temperature of 98.2F (36.8C). His blood pressure is 72/40 mm Hg. His oxygen satura on, measured by pulse oximetry, is 98% while brething room air. Despite the abnormal vital a signs, the baby does not appear distressed or even uncomfortable. He is moving all extremities, and he opens his eyes and looks around. His oropharynx is clear, with no visible foreign bodies. There is no rhinorrhea or nasal congestion. His lung sounds are clear, and despite the significant tachypnea, no retractions, grunting, or nasal flaring are present. On auscultation of the heart, rapid heart sounds are noted, and as a result of the tachycardia, an assessment for murmurs is not possible. His distal extremities are pink and have a normal capillary refill.While placing an intravenous line and connecting the child to a monitor, an electrocardiogram (EKG) is obtained.

alm cq sd ow

Figure 1

nlo

ad .bl og

An 8-day-old boy is brought into the emergency department (ED) by his parents for a chief complaint of rapid breathing. During the past several hours, the patient's mother noted an increased respiratory rate and decreased feeding ability. The child had previously fed for 10 minutes at each breast every 2 hours, but he is now crying and unable to latch on for more than a few seconds at each breast. Both parents state that they think the patient looks pale. Up until the day of presentation, the patient had been doing well, with no prior episodes of difficulty breathing. He was born full-term via normal spontaneous vaginal delivery, without complications during the pregnancy or delivery. He has had no sick contacts and has not been in day care. He has not had a fever, cough, or runny nose. His urine output has been normal, and he has had 2-3 nonbloody bowel movements daily. Other than spitting up a small amount after feedings, there have not been any episodes of vomiting. The parents state that he seems increasingly fussy. There have been no episodes of apnea, cyanosis, choking, or gagging, as well as no discernible changes in muscle tone.

sp ot. c

om

AN 8-DAY-OLD BOY WITH TACHYPNEA AND DIFFICULTY FEEDING

3o clinical cases from eMedicine

What type of arrhythmia is shown on the EKG?

o o o o

Torsade de pointes Atrial fibrillation Supraventricular tachycardia Ventricular tachycardia

ww w. me

dic

alm cq sd ow

nlo

ad .bl og

sp ot. c

Hint: Consider the morphology of the QRS complexes (narrow vs wide) and the timing of the complexes (regular vs irregular).

om

3o clinical cases from eMedicine o Case 26 Answer

Discussion:

This patient was diagnosed with supraventricular tachycardia (SVT), which is defined as a regular, rapid rhythm that requires only atrial or atrioventricular tissue for its initiation and maintenance. Paroxysmal SVT (PSVT) is the most common dysrhythmia in children. It has an interna onal prevalence of about 2 cases per 1,000 people. PSVT tends to manifest in infancy and early childhood. The presentation of PSVT is quite variable in children, ranging from an incidental finding in an asymptomatic resentation patient to fulminant cardiogenic shock. Infants present with caretakers complaining of rapid breathing, poor feeding, sweating with feeding, pallor, lethargy, and excessive crying. Older children may complain of chest lethargy, pain, shortness of breath, and palpitations. The physical examination is likewise variable, depending upon the child's age and heart rate and on the duration of the episode (which can last from seconds to days). For infants, the examination is remarkable for a regular tachycardia. Normal resting heart rates for neonates can be up to 160 beats per minute. The upper limit of the normal heart rate decreases with age un l late adolescence, when children's heart rates are similar to those of adults. Heart rates ranging beyond the normal upper limits, par cularly in excess of 240, should be highly suspicious for SVT. In addi on to tachycardia, infants may have physical findings of pallor, irritability, lethargy, tachypnea, weight loss (or failure to gain), poor perfusion, weak bility, pulses/hypotension, hepatomegaly, and, sometimes, cardiogenic shock. A pounding sensation in the neck may be caused by cannon A waves, which occur when the atrium contracts at the same time as the ventricle. Older children typically have benign examinations (except for the findings of tachycardia and tachypnea). There are 3 types of SVT: (1) atrial tachycardia (ectopic, or nonreciproca ng, atrial tachycardia), (2) atrioventricular nodal reentrant tachycardia (AVNRT), and (3) atrioventricular reentrant (or reciproca ng) ricular tachycardia (AVRT). In the United States, reentrant tachycardias are the most common cause of PSVT in the pediatric population, with AVRT being more common than AVNRT. AVRT consists of 2 or more func onally (and, than usually, anatomically) distinct pathways between the atria and ventricles. The first pathway is usually the atrioventricular (AV) node. The second is an accessory pathway that may be an anatomically separate bypass tract between the atrium and ventricle (such as the bundle of Kent). Wolff Parkinson-White (WPW) syndrome Wolff-Parkinson preexcitation is a good example of SVT caused by an anatomically separate bypass tract. Each pathway has different electrophysiologic characteristics; one pathway is fast (a short conduction time and a long refractory period), while the other is slow (a longer conduction time and a shorter refractory period). In AVNRT, both pathways exist in the AV node itself. In AVNRT, for example, while the child is in regular sinus, a premature while atrial beat may block in the fast pathway (because of its longer refractory period). Since the fast pathway is blocked, the signal conducts down the slow pathway. Then, when the impulse reaches the insertion of the fast pathway, which has now recovered after its refractory period, the impulse conducts in a retrograde fashion through the fast pathway. This results in a circuit loop tachycardia, with an impulse moving in a loop down the slow pathway and up the fast one. Other causes of PSVT include sympathomimetic stimulation (such as medications for upper respiratory infection), structural defects, and atrial ectopy. About half of all SVT cases occur without underlying heart disease; these cases are termed idiopathic. Idiopathic SVT is more common in younger patients than in older idiopathic.

ww w. me

dic

alm cq sd ow

nlo

ad .bl og

Diagnosis:Supraventricular tachycardia Supraventricular

sp ot. c

om

AN 8-DAY -OLD BOY WITH T ACHYPNEA AND DIFFICULTY F EEDING

3o clinical cases from eMedicine


children. WPW syndrome preexcita on accounts for 1020% of cases. Congenital heart defects may also predispose children to SVT. Children who have undergone cardiac surgery are also more prone to developing this arrhythmia. The diagnosis of PSVT is made based on patient history, physical examination findings, and EKG findings. The EKG ndings include an excessively rapid (usually between 200 and 280 bpm) regular tachycardia, o en without discernible P-waves preceding the QRS complexes. P-waves, when visible in PSVT, may have an abnormal axis and may be seen within or following the QRS complexes. In most cases, the tachycardia will be narrow complex; however, wide-complex PSVT is possible if aberrancy is present or if the AVNR conduction occurs in an antidromic fashion with WPW syndrome. It is important to distinguish PSVT from sinus tachycardia, which is the most common tachycardia in children. Sinus tachycardia and PSVT have dramatically different treatments. Laboratory evaluation will depend on the clinical scenario, but practitioners should consider checking electrolytes, thyroid function, and hemoglobin level. Emergency management of children with PSVT should start with a focus on first assessing the airway for potential compromise or obstruction, then evaluating breathing and ventilation, and assessing the circulatory status of the patient (ie, the "ABCs"). When evaluating the circulatory status, unstable patients should undergo immediate synchronized cardioversion with 0.5 J/kg. The power should be increased as needed to 2 J/kg. If the cardioversion fails, overdrive pacing is another option. For stable patients, vagal maneuvers can be attempted first. Infants possess a diving reflex, in which vagal tone will increase in response to a cold stimulus (eg, ice) on the face. In older children, unilateral carotid massage, eyeball pressure, or even a headstand is more likely to cause conversion. When these maneuvers are not typically successful in converting the rhythm, medication is usually required. Adenosine, given at a dose of 0.1 mg/kg rapid intravenous push, is the rst-line agent. This may be repeated in doses of 0.3 mg/kg, as needed. Amiodarone and procainamide are also safe in children with SVTs refractory to adenosine. In general, adenosine and digoxin should be avoided in situations where there is a suspicion of WPW syndrome, as they may potentiate conduction through the accessory pathway, resulting in increased ventricular rates and possible degeneration to ventricular fibrillation. Children without WPW syndrome preexcita on can be maintained on chronic oral propranolol or verapamil (in children older than 5 years), if needed after the acute illness. These agents should be used in consultation with a pediatric cardiologist. Disposition of children with PSVT depends on the clinical circumstance. A child in shock or with concerning comorbidities should be admitted to a pediatric intensive care unit (ICU). Asymptomatic children without frequent recurrence may be able to follow up with a cardiologist, without initiating medications. For younger infants, the admission threshold should be lower. The patient in this case failed cardioversion with vagal maneuvers but converted after two boluses of adenosine, which is consistent with a reentrant circuit involving the AV node. The patient was admitted to a monitored pediatric unit and evaluated by pediatric cardiology. The patient was initially discharged on digoxin; however, after further follow-up, the medication was discontinued, because the child's cardiologist came to suspect WPW syndrome as the likely etiology for the PSVT.

ww w. me

dic

alm cq sd ow

nlo

ad .bl og

sp ot. c

om

3o clinical cases from eMedicine Case 27

Background

ww w. me

dic

On physical examina on, the pa ent is afebrile and has a pulse of 72 bpm, a blood pressure of 130/82 mm Hg, a respiratory rate of 12 breaths/min, and a normal oxygen satura on while breathing room air.He is welldeveloped and well-appearing. Examination of the anterior neck reveals a nontender, nonerythematous, uctuant mass measuring approximately 10 8 cm in the midline of the lower neck, with slight extension to the right side of the midline. The mass moves up and down when the patient swallows, and it slightly displaces anteriorly with protrusion of the tongue. No cervical lymphadenopathy is appreciated. The lung fields are clear bilaterally, without any evidence of stridor or wheeze. The heart has a regular rate and rhythm, without murmurs, and the abdomen is soft and nontender, without evidence of masses. The cranial nerves are intact, and the remainder of the neurologic exam is unrevealing as well.

alm cq sd ow

nlo

Figure 1

ad .bl og

45-year-old man presents to his primary care physician (PCP) complaining of gradual swelling in his anterior neck over the past 6 months. At rst, he thought nothing of the swelling, expec ng it go away on its own; however, over the past 2 months, it has become more no ceable. The pa ent has becom e concerned that the swelling may be caused by cancer. He has not experienced any pain in the area of the swelling, nor has he experienced fevers. Additionally, he denies any difficulty in swallowing and any alteration of his voice. There are no problems with his breathing. The patient has no history of trauma, and he denies any significant personal medical history. The family history is unremarkable. He is not currently taking any medications, he does not smoke, and he does not use any illicit substances. He is a social drinker.

sp ot. c

om

INSIDIOUS SWELLING IN THE NECK OF A 45-YEAR-OLD MAN

3o clinical cases from eMedicine


Some routine laboratory blood tests, which include a complete blood cell (CBC) count and an electrolyte panel, are sent, as well as a rapid assay for thyroid function tests. All of the laboratory investigations, including the thyroid studies, are within normal limits. An ultrasound of the neck is obtained (Figure 1). As a follow-up, a computed tomography (CT) scan of the neck is also performed for further evalua on (Figure 2).

What is the diagnosis?

Hint: This is the most common congenital anomaly resulting in a midline neck mass. o o o o Cystic vascular abnormality Cervical teratoma Ectopic thyroid Thyroglossal duct cyst

ww w. me

dic

alm cq sd ow

nlo

Figure 2

ad .bl og

sp ot. c

om

3o clinical cases from eMedicine o Case 27 Answer

Discussion:

On radiologic images, thyroglossal duct cysts appear as a cystlike mass along the course of the thyroglossal duct. l They must be differentiated from dermoid cysts and lymphangiomas. A dermoid cyst usually contains fat; lymphangioma is most common in infancy or early childhood, and it usually occurs in the posterior triangle of usually the neck, behind the sternocleidomastoid muscle. A thyroglossal duct cyst must also be differentiated from an ectopic thyroid gland. If an ectopic thyroid gland is mistakenly removed, the patient may require long long-term thyroid treatment for hypothyroidism. Often, patients with an ectopic thyroid also have hypothyroidism, and they will have an elevated thyroid-stimulating hormone (TSH) level. stimulating The diagnosis of thyroglossal duct cyst is made on the basis of the clinical history and confirmed with diagnostic imaging. Most patients with thyroglossal duct cysts present with either a history of a slowly growing, asymptomatic mass or of a relatively r rapid-growing mass (if the cyst is infected) in the anterior midline of the growing neck. Frequently, the swelling is exacerbated during an upper respiratory infection. The pathognomonic sign of

ww w. me

The differential diagnosis for neck masses can be categorized by the location of the mass itself; the usual categorization is between lateral and midline masses. The most frequent causes of lateral masses are lymphadenopathy, branchial cleft cyst malignancy, cystic lymphangioma, and dermoid and terato cysts. teratoid Although thyroglossal duct cysts are the most common etiology for midline masses, the differential diagnosis also includes dermoid and teratoid cysts, ectopic thyroid tissue, malignancy, and cystic lymphangiomas.

dic

alm cq sd ow

The cysts usually appear in the midline and can be present anywhere along the line of fetal descent from the foramen cecum to the level of the thyroid gland. From an embryologic perspective, the thyroid gland develops during the third week of life as an outgrowth of the floor of the primitive pharynx. The primitive thyroid then g descends from the foramen cecum to its mature position in the anterior neck through the thyroglossal duct. The thyroglossal duct is normally resorbed by 7 to 10 weeks of fetal life. Abnormal persistence of the thyroglossal tract accompanied by mucus production from the endothelial lining of the tract leads to the development of a thyroglossal duct cyst. Approximately 7% of the popula on has thyrogloss duct remnants, and the distribution al is equal among males and females. The cysts are usually found in children or adults younger than age 30 years, but they can develop in adults of any age. In recent years, a number of older patients, including patients in their 80s and 90s, have presented with thyroglossal duct cysts. There are 4 general types of thyroglossal duct cysts: thyrohyoid (61% of cases), suprahyoid (24%), suprasternal (13%), and intralingual (2%).

nlo

The ultrasound image in Figure 1 shows a large cys c mass anterior to the thyroid gland (arrowheads). The contrast-enhanced CT scan (Figure 2) demonstrates that same predominantly midline cystic mass extending enhanced anteriorly to the thyroid gland and under the strap muscles, without any evidence of ectopic thyroid tissue. The findings are consistent with the diagnosis of a thyroglossal duct cyst. This diagnosis is the most common etiology for a midline neck mass. Thyroglossal duct cysts usually occur between the hyoid bone and the thyroid gland, and they represent up to 70% of congenital neck anomalies. Thyroglossal duct cysts are second only to lymphadenopathy as the most common cause of a neck mass. ost

ad .bl og

Diagnosis: Thyroglossal duct cyst

sp ot. c

om

INSIDIOUS SWELLING IN THE NECK OF A 45 EAR-OLD MAN 45-Y

3o clinical cases from eMedicine


a thyroglossal duct cyst is that it moves with swallowing and with protrusion of the tongue; however, the mobility of larger cysts may be restricted. Imaging studies, including ultrasonography and CT scanning of the neck, will confirm the diagnosis and help to rule out the presence of a possible ectopic thyroid gland. Thyroid function tests should be obtained to confirm normal thyroid function. Once diagnosed, thyroglossal duct cysts are removed because they are cosmetically undesirable and have the potential to become infected and undergo malignant transformation. The treatment of choice is the Sistrunk procedure, named a er Dr. Walter Ellis Sistrunk and rst described in an ar cle in 1920. The procedure, rather than involving a simple excision of the cyst, involves dissecting the central portion of the hyoid bone, with extension of the excision up to the base of the tongue to include excision of a small block of muscle around the foramen cecum. Because of the increased risk of thyroglossal duct carcinoma, some practitioners further recommend the addition of thyroid suppression therapy or a complete thyroidectomy; however, this practice remains controversial. The recurrence rate associated with simple excision of a thyroglossal duct cyst is approximately 50%, whereas the recurrence rate associated with a formal Sistrunk procedure is approximately 5%. The rate of recurrence a er a Sistrunk procedure is increased, however, when a thyroglossal duct is ruptured during dissection. A history of previous infection of the cyst, previous incision and drainage procedures, and adherence of the cyst to the skin all increase the risk of rupture during dissection. If the cyst is infected at the time of diagnosis, treatment with antibiotics, such as ampicillin/sulbactam, amoxicillin/clavulanate, or clindamycin, is indicated before surgical excision. The most common complications of thyroglossal duct cysts are infection with the possibility for abscess formation, spontaneous rupture, and formation of a secondary sinus tract. A Sistrunk procedure mistakenly performed for thyroid ectopia that removes thyroid tissue can cause hypothyroidism. The cysts can compress the trachea and lead to respiratory distress, especially if they are rapidly expanding (although this is not common). Carcinoma is the most feared complica on, occurring in about 1% ofall cases, with papillary carcinoma accoun ng for 8592% of malignancies and follicular carcinoma accoun ng for the rest. Most patients who develop carcinoma tend to present at a later age. Cancer in a thyroglossal duct cyst seems to be more common in females than in males. The diagnosis of carcinoma arising in a thyroglossal duct cyst is typically made postoperatively by histology. Because the patient in our case presented with a thyroglossal duct cyst which (on clinical grounds) was not infected, he was not given antibiotics. The ultrasound study confirmed the presence of a thyroglossal duct cyst and ruled out the possibility of an ectopic thyroid gland. The patient underwent an elective Sistrunk procedure, with no rupture during dissection. There were no complications. He was discharged from the hospital the following day. Postoperative histologic analysis did not reveal any evidence of malignancy. On follow-up in the clinic 2 weeks later, the pa ent was noted to be doing well.

ww w. me

dic

alm cq sd ow

nlo

ad .bl og

sp ot. c

om

3o clinical cases from eMedicine Case 28

Background

dic

On physical examina on, the pa ent is afebrile, with a pulse of 65 bpm, a blood pressure of 120/84 mm Hg, and a respiratory rate of 15 breaths/min. His room air satura on reading is 100%. In general, he is well -appearing and in no acute distress. The patient's neck examination shows no jugular venous distention. The heart sounds, including S1and S2, reveal no audible murmurs, rubs, or gallops. The apical impulse is nondisplaced and of normal impact. The lung sounds are diminished throughout, but there are no wheezes, rales, or rhonchi. He has no edema of the lower extremities, and the distal pulses are easily palpable. All other exam findings, including a neurologic examination, are unremarkable. The pa ent is placed on a cardiac monitor, and an 18gauge intravenous (IV) catheter is inserted into the antecubital fossa. Laboratory tests consisting of a complete blood count (CBC) and serum electrolytes are ordered. A portable chest radiograph reveals slight hyperinflation and hyperlucency of the lung fields, with a flattened diaphragm and central pulmonary artery enlargement. An electrocardiogram (ECG) is obtained (see Figure 1).

ww w. me

alm cq sd ow

Figure 1

nlo

ad .bl og

A 24-year-old man with no significant past medical history presents to the emergency department (ED) with a complaint of several episodes of a sensa on of nearly blacking out. The episodes have occurred about 34 mes over the 3 days before presenta on. The dura on of each episode has ranged from a few minutes to over an hour. The patient notes that he has felt his "heart beating really fast," with associated light-headedness. He denies having any chest pain, shortness of breath, or nausea associated with these events. He cannot identify exacerbating or alleviating factors; specifically, he denies exertion as an inciting factor. The remainder of his review of systems is negative except for some mild chronic shortness of breath. The patient takes no medica ons at home and has no ac ve medical condi ons. He smokes 2 -4 packs of cigare es per day and has done so for 5-6 years. He denies any illicit drug use or recent use of over-the-counter medications or herbal remedies. He has no history of any significant cardiac disease or sudden cardiac death in his family.

sp ot. c

om

NEAR-SYNCOPE IN A 24-YEAR-OLD MAN

3o clinical cases from eMedicine


What is the diagnosis? Hint: Pay close attention to the intervals and the QRS complex morphology. o o o o Wolff-Parkinson-White syndrome Ventricular fibrillation Sinus tachycardia Non-sustained ventricular tachycardia

ww w. me

dic

alm cq sd ow

nlo

ad .bl og

sp ot. c

om

3o clinical cases from eMedicine o Case 28 Answer

Discussion:

ww w. me

dic

WPW syndrome can be identified by a classic fusion QRS complex ECG pattern that is a combination of simultaneous normal conduction through the AV node and aberrant conduction through the accessory tract. This fusion QRS complex leads to par cular ECG features that include a shortened PR interval (<120 msec) and a widened QRS complex with a delta wave representing preexcitation of the ventricle through the accessory pathway. The distinctive ECG pattern of the accessory pathway was initially described by Wolff, Parkinson, and initially White in 1930 as a bundle branch block with a short PR interval. Addi onally, as men oned, WPW syndrome is recognized as the most common form of ventricular preexcitation, although it likely represents a collecti of collection pathologic conditions rather than a single structural abnormality.

alm cq sd ow

Figure 1

nlo

Preexcitation is characterized by an accessory pathway within the heart that conducts action potentials between the atria and ventricles outside of the normal conduction system (which conducts through the atrioventricular [AV] node-His-Purkinje system). The phenomenon was dened by Durrer et al in 1970, who Purkinje stated that "preexcitation exists, if in relation to atrial events, the whole or some part of the ventricular muscle is activated earlier by the impulse originating from the atrium than would be expected if the impulse reached pulse the ventricles by way of the normal specific conduction system only. Of the various types of preexcitation syndromes, the most common is Wolff Wolff-Parkinson-White (WPW) syndrome.

ad .bl og

Diagnosis: Wolff-Parkinson-White syndrome White

sp ot. c

om

NEAR -SYNCOPE IN A 24-YEAR -OLD MAN

3o clinical cases from eMedicine


Normal cardiac conduction of action potentials from the atria to the ventricles occurs exclusively through the AV node; the atrial impulses are subsequently propagated through a specialized conduction system (the AV-HisPurkinje system) and finally terminate in the ventricular myocardium. Action potential conduction through the AV node depends on slow inward calcium currents. In addition, the AV nodal system exhibits decremental conduction, which provides a protective effect; as the cardiac cycle is shortened (eg, the heart rate increases), there is decreased conduction through the AV node. This phenomenon limits the ventricular response to rapid atrial rates, such as those observed in atrial fibrillation or atrial flutter. In preexcitation syndromes such as WPW, however, the action potential conducts to the ventricles at least partially through an accessory pathway termed the AV bypass tract or the bundle of Kent. Action potential propagation in the accessory pathway in WPW syndrome occurs through a rapid cellular influx of sodium. The consequence of the sodium-dependent action potential propagation mechanism is an accelerated conduction of impulses by the accessory bypass tracts, which leads to early activation of the ventricle as demonstrated by a shortened PR interval and a "slurred" QRS complex (ie, delta wave). Ventricular depolarization slowly spreads out from the bypass tract, while normal conduction that has been somewhat delayed through the AV node begins to conduct through the His-Purkinje system and spreads quickly to the remaining ventricular musculature. Although conduction velocity through the accessory pathway is faster than it is through the AV node, the accessory pathway often has a longer refractory period and, as such, is slower to recover excitability. Interestingly, the conduction of action potentials through the accessory pathway is nondecremental; therefore, the protective effect achieved by the AV node at higher heart rates is lost. These differences have important clinical implications. For example, a premature beat may conduct through the AV node normally while the accessory pathway remains refractory to conduction. The impulse then travels in a retrograde direction through the accessory pathway after ventricular depolarization, when it has recovered excitability. The consequence of this is the propagation of a reentry loop termed an orthodromic AV reciprocating tachycardia. This can then lead to rapid ventricular response rates that can degenerate into ventricular tachyarrhythmias. Rarely, antidromic tachycardias occur; conduction occurs in an anterograde direction through the accessory pathway and in a retrograde direction through the AV node. Ventricular depolarization occurs through both the AV node-His bundle pathway and the accessory pathway; each pathway affects the ventricles by various degrees, depending on their relative activation times. As AV nodal conduction is delayed by either rapid atrial pacing or premature atrial beats, the accessory pathway contributes to a greater degree, resulting in a wider QRS morphology with an increasingly slurred delta wave. If the relative conduction time through the AV node is sufficiently delayed, total activation of the ventricle may occur through the accessory pathway. The presence of accessory bypass tracts is not uncommon in the general population; however, less than half of the people with bypass tracts actually sustain a tachyarrhythmia. WPW syndrome affects approximately 0.150.2% of the general popula on, and of these individuals, 6070% have no other evidence of heart disease. Mortality and morbidity associated with WPW syndrome occur as a result of associated dysrhythmias or from mistreatment of these dysrhythmias with inappropriate medications. Most studies report that the incidence of sudden death is in the 0-4% range. Men are aected more o en than women, accoun ng for 60 -70% of all cases. Although this disease affects people of all ages, it is typically first recognized in children and young adults who present to the ED or their primary care physician with symptoms secondary to a dysrhythmia. Genetic mutations have been identified (by mapping genetic defects to specific loci) that account for the increased incidence of WPW syndrome in certain families. In patients with suspected WPW syndrome, evaluation should initially be directed at confirming the diagnosis and recognizing any potentially life-threatening arrhythmias. In patients with life-threatening arrhythmias, direct-current cardioversion should be immediately administered. In stable patients with tachyarrhythmias, an antiarrhythmic medication may be administered to terminate the arrhythmic episode, rather than immediately performing electrical cardioversion. Studies have demonstrated that the best and most cost-effective treatment for patients with asymptomatic WPW syndrome is simple observation. Most patients with symptomatic arrhythmias, drug-refractory WPW syndrome, or significant life-threatening arrhythmias are treated with nonpharmacologic therapy.

ww w. me

dic

alm cq sd ow

nlo

ad .bl og

sp ot. c

om

3o clinical cases from eMedicine


Surgical ablation, previously the standard technique for drug-refractory WPW syndrome, has been replaced by catheter-based procedures. Compared with surgical techniques, catheter ablation has comparable success rates, lower mortality and complication rates, and improved cost-effectiveness. Moreover, newer catheter mapping systems now allow shorter procedure times. Surgical ablation, however, may be necessary in patients in whom catheter ablation has failed. Because this patient had a symptomatic tachyarrhythmia, he underwent electrophysiologic mapping followed by transvenous catheter ablation. He has remained asymptomatic since this procedure. Patients with infrequent or minimally symptomatic arrhythmias may be treated pharmacologically. The aim of pharmacologic therapy is to alter the electrophysiologic properties, such as the refractoriness or conduction velocity of the AV node or the accessory bypass tracts.

ww w. me

dic

alm cq sd ow

nlo

ad .bl og

sp ot. c

om

3o clinical cases from eMedicine Case 29

Background

dic

On evaluation in the emergency department, the patient was asymptomatic except for a persistent cough with clear sputum. She was a nonsmoker and had no previous pulmonary disease. She did not have a history of tuberculosis or known exposure to risk factors. A purified protein derivative of tuberculin (PPD) test performed 3 months ago yielded nega ve resul She has not been taking any drugs and has no known allergies. Physical ts. examination revealed a well-appearing woman in no distress with a respiratory rate of 16 breaths per minute, a temperature of 35.9C, a blood pressure of 110/60 mm Hg, and a heart rate of 95 beats per minute. Her lungs were clear, with no wheezing, rhonchi, or rales. Her heart sounds were normal, with no murmurs. The remainder of her examination yielded unremarkable findings.

ww w. me

alm cq sd ow

Figure 2

nlo

ad .bl og

A 33-year-old woman was evaluated because of persistent pulmonary inltrates. She was well un l 7 months ago, when she developed fever, a productive cough, and left-sided pleuritic chest pain. A chest radiograph showed an infiltrate in the left lower lobe. Several courses of antibiotics led to an improvement of her symptoms, though a dry cough persisted. Subsequent chest CT showed attenuating airspace disease in the left lower lobe with air bronchograms and ill-defined nodules in both lungs. Before her current presentation, she underwent 2 bronchoscopic procedures, with transbronchial biopsy performed during the most recent one. All bacterial cultures were negative, and cytology and pathology failed to reveal malignancy. Given the lack of an infectious etiology, corticosteroids were started. However, the patient's condition did not improve.

sp ot. c

om

NONRESOLVING PNEUMONIA IN A YOUNG WOMAN

3o clinical cases from eMedicine

Chest CT was performed (see Image). Positron emission tomography (PET) showed increased uptake in the left posterior por on of the lower lung with a standard uptake value (SUV) of 6.5 and no uptake in the nodules or hilar or mediastinal nodes.

What is the diagnosis? Hint

The patient's indolent presentation is typical of this condition.

ww w. me

dic

alm cq sd ow

Figure 3

nlo

ad .bl og

sp ot. c

om

3o clinical cases from eMedicine Case 29 Answer

Diagnosis: Bronchoalveolar carcinoma (BAC) Discussion:

Biopsies of the left lower and upper lobes of the lung were performed by means of video-assisted thoracoscopic surgery (VATS). The results indicated a stage IV BAC. Despite serial chemotherapy over 4 years, the disease progressed (see Image 2). The pulmonary inltrates expanded and became cavitated. The patient's respiratory status worsened, and she became oxygen dependent at rest. Recurrent pulmonary infections and airway obstruction secondary to tumor progression resulted in progressive respiratory failure, which ultimately led to her death. Lung cancer is the most common cancer worldwide and has the greatest mortality rate. Lung adenocarcinoma is the most common histologic form of lung cancer (31.5%). Its frequency is rising in women and in nonsmokers. BAC is a subtype of lung adenocarcinoma and has a wide variety of distinct clinical manifestations, which often lead to an ini al misdiagnosis. It is rela vely rare (accoun ng for 2.6% of all the lung cancers), and it is seen mainly in women aged 40-70 years. It typically has a rela vely indolent course, an its association with smoking d is weaker than that of other forms of lung cancer. In 1999, the World Health Organiza on (WHO) International Association for the Study of Lung Cancer (IASLC) classification defined BAC as an adenocarcinoma with a lepidic growth pattern (growth along intact alveolar septa) without invasive growth, such that the alveolar architecture remains intact. Patients with BAC may present with one or several nodules or with diuse parenchymal inltrates. More than 50% of BACs may be associated with focal scars. A final diagnosis of BAC can be achieved only by analyzing a surgical specimen. Metastatic adenocarcinomas can mimic BACs; therefore, a history of an extrathoracic adenocarcinoma precludes the diagnosis of BAC. The 2 main histologic types of BAC are mucinous and nonmucinous. The mucinous type (41-60%) has a colloidlike histologic appearance because of mucin that fills the adjacent alveoli. In clinical terms, this finding manifests as the expectoration of mucoid material. The mucinous form tends to be multicentric and often causes lobar consolidation resembling that of pneumonia, and it worsens the patient's prognosis. Whether the multicentricity is due to aerogenous or lymphatic spread or the expression of several independent malignant clones is still debated. The nonmucinous type (21-45%) is composed of cuboidal or columnar cells. It is most likely to result in a solitary nodule and improves the prognosis. Approximately 12-14% of BACs are of a mixed subtype, and as many as 7% are indeterminate. About 50-60% of pa ents are asymptoma c in the early stages, and the diagnosis may be made incidentally during chest radiography. Cough (35%), sputum (24%), and shortness of breath (15%) are usually seen in only advanced disease. Weight loss, hemoptysis, and fever occur less frequently than these other findings. Bronchorrhea (5%) is a late sign and most o en associated with the diuse, mucinous form. Diuse lling of the alveolar spaces by mucin causes severe hypoxia, which is also a manifestation of late disease. The radiologic presenta on of BAC varies, ranging from solitary (43%) or mul ple (27%) nodules to diuse airspace disease (30%). The CT a enua on of these forms ranges from that of so ssue (frank inltrate) to ground-glass opacity (hazy shadow). A solitary pulmonary nodule is typically peripheral, slowly growing, and unlikely to evolve to widespread disease. Airspace disease (pneumonic form) can be indistinguishable from pneumonia and is due to a combination of tumor cells and mucin. Multiple nodules are often bilateral and sometimes mimic pulmonary metastatic disease. In rare cases, BAC can manifest as lobar atelectasis or as cysts and cavi es (7%).

ww w. me

dic

alm cq sd ow

nlo

ad .bl og

sp ot. c

om

NONRESOLVING PNEUMONIA IN A YOUNG WOMAN

3o clinical cases from eMedicine


Certain radiologic signs should raise a clinician's suspicion for BAC: air bronchograms (open bronchus in an area of infiltrate), pseudocavitation (low-attenuation regions in a nodule), or a CT angiographic sign (visualization of vessels in regions of consolidation after intravenous administration of contrast material). PET is often used to evaluate BAC. BAC usually has an SUV lower than that of other lung cancer subtypes; the decreased value is due to low metabolic activity or low cellularity. In general, negative PET results are associated with focal disease, and increased uptake is associated with diffuse and poorly differentiated forms. The differential diagnosis of the pneumonic form of BAC includes all the infectious causes of nonresolving pneumonia, other primary or metastatic lung malignancies, lymphoma, sarcoidosis, pulmonary alveolar proteinosis, diffuse alveolar hemorrhage, vasculitis, eosinophilic pneumonia, bronchiolitis obliterans organizing pneumonia (BOOP), or pulmonary edema.

This case involved a rare presentation of the pneumonic form of BAC in a young patient with a poor response to treatment. BAC has clinical, histologic, and radiographic characteristics distinct from those of other nonsmall cell lung cancers. Being familiar with the multiple forms of presentation of BAC can help in the early diagnosis and subsequent treatment of a potentially curable disease.

ww w. me

dic

alm cq sd ow

nlo

No optimal therapy has been defined for recurrent or advanced BAC. The response rate to conventional chemotherapy is low (14% response rate to a 96-h infusion of paclitaxel). Recent studies showed improved results with epidermal growth factor (EGFR) or tyrosine kinase inhibitor. Women without a history of smoking appear to respond most o en to ge nib. The reported response rate is 12 -19% for ge nib and 26% for erlotinib. Somatic mutations of EGFR appear to make the BAC responsive to these targeted inhibitors in approximately only 10% of pa ents. Surgical op ons include cura ve resec on for limited forms (usually up to 3 separate nodules), palliative resection, or, potentially, lung transplantation.

ad .bl og

The mucinous type, diffuse lesions, and symptomatic disease, are associated with a worsened prognosis. Pa ents with disease in the T1N0M0 stage have a 90.5% 5year survival rate, as opposed to those with diffuse bilateral disease, who have 0% survival at 5 years. A 100% 5-year survival rate is reported for BAC with a >75% lepidic growth pa ern and a central scar <5 mm or an intact elas -fiber framework. c

sp ot. c

om

3o clinical cases from eMedicine Case 30

Background

Figure 4

Hint Check out those ST segments!

ww w. me

dic

alm cq sd ow

nlo

ad .bl og

sp ot. c

The patient was resuscitated from cardiac arrest caused by ventricular fibrillation. What is the diagnosis?

om

RESUSCITATION FROM VENTRICULAR FIBRILLATION ARREST

3o clinical cases from eMedicine Case 30 Answer

Diagnosis: Acute anterior-wall myocardial infarction Discussion:

Note the hyperacute T waves in the anterior leads and the elevated ST segments in these leads. Although the eleva ons are not convex in many leads, they are in leads V1 and aVL. Furthermore, the recipocal ST-segment r depressions in the inferior leads confirm the diagnosis.

ww w. me

dic

alm cq sd ow

nlo

ad .bl og

sp ot. c

om

RESUSCITATION FROM VENTRICULAR FIBRILLATION ARREST

S-ar putea să vă placă și